SlideShare una empresa de Scribd logo
1 de 129
Neurology
Archer USMLE Step3 Reviews

www.CcsWorkshop.com












Risk Factors - Stroke

Age >50
Family history of CAD, CVD, or PVD before age 60
Clinical manifestations of CAD or PVD
Hypertension
Diabetes mellitus
Elevated cholesterol
Smoking
Hyperhomocysteinemia
Carotid bruit ( presene only if luminal stenosis >50%)
History of TIA
History of paroxysmal or persistent atrial fibrillation






Primary Prevention Of Stroke

Control hypertension  Use ACE
inhibitor in high-risk patients  if they
are hypertensive or diabetic, or if they
have already had a vascular ischemic
event.
Smoking cessation.
Achieve glycemic control in diabeteics
Start warfarin in most patients with
nonvalvular atrial fibrillation in whom
sinus rhythm cannot be restored,
including (CHADS2)




High-risk patients with previous embolization
(TIA/CVA), age > 75, HTN, DM Type II and
those with congestive heart failure or
decreased left ventricular function
Elderly patients to maintain an INR between
1.8 and 2.5
Primary Prevention Of Stroke








Do not use warfarin  in patients who cannot have
adequate monitoring of INR, cannot receive medications
in a predictable fashion, or have increased risk for
bleeding (i.e., those with a high risk of traumatic fall
(elderly), those with high-risk occupations, or those who
have a bleeding disorder).
Start antiplatelet therapy in Afib for prevention for
storke- ASA 50 to 325 mg po qd, Clopidogrel, 75 mg po
qd, or Aggrenox
- in low-risk patients with nonvalvular atrial
fibrillation (lone afib)
- in higher-risk patients who decline
anticoagulation, would not comply with INR monitoring, or
are at very high risk for bleeding.
Discontinue combination estrogen and progestin
treatment in healthy postmenopausal women.
Consider starting low-dose every-other-day aspirin in
otherwise asymptomatic women over the age of 45.
Primary Prevention - Stroke


Carotid endarterectomy should be
considered in asymptomatic
patients with >60% stenosis  who
have no other contraindications for
surgery (especially severe
cardiopulmonary disease), are
under age 75, and are expected to
live >5 years .
Stroke - Management

Immediate Neurologic Assessment (<25 minutes)
 Alert Stroke Team of possible Thrombolytic. Determine onset of CVA
symptoms  Consider Thrombolytics within 3 hours of onset
 Physical Examination  Neurologic Examination  Assess Level of
Consciousness (Glascow Coma Scale) & Assess Stroke Severity
Step 2: Rule-out Hemorrhagic CVA
 Imaging





If Head CT suggests intracranial bleeding  Neurosurgery consultation




Reverse anticoagulants or Bleeding Disorder and Manage Hypertension
appropriately

Head CT negative despite high suspicion for SAH




Obtain urgent noncontrast Head CT (<25 minutes)  Head CT read by
radiologist (<45 minutes)
Lateral Neck XRay Indications  Altered Level of Consciousness , Trauma

Obtain Lumbar Puncture to assess subarachnoid blood  remember,
Lumbar Puncture contraindicates Thrombolytics if you are
suspecting an ischemic cva also.

Head CT negative suggesting Ischemic CVA


Give Thrombolytic Therapy if meets the criteria/ If not, just ASA or
Aggrenox.
Stroke - Management
Step 3: Thrombolytic Therapy (if
indicated)
 Know Thrombolytic Contraindications


Know indications for Thrombolytic therapy



Persistent neurologic deficits
CVA Symptom onset <3 hours prior
Stroke - Thrombolytics

Inclusion Criteria

Age over 18 years

Clinical Diagnosis of acute Ischemic Stroke

CT Head compatible with Ischemic CVA diagnosis

Known time of onset under 3 hours before Thrombolytics


Do not use intravenous tPA beyond 3 hours of symptoms

Exclusion Criteria

Improving or mild neurologic deficit

Seizure at onset

Head Injury or CVA within 90 days

History of Intracranial Hemorrhage

Suspected Subarachnoid Hemorrhage (SAH)  Hemorrhage on CT Head or History
suggests SAH even despite negative Head CT (worst headache/ photophobia)

Hypertension refractory to antihypertensives Systolic Blood Pressure over 185 and
Diastolic Blood Pressure over 110 ( if elevated, control and then give tpa)

Major surgery in last 14 days

Gastrointestinal hemorrhage in last 21 days

Genitourinary hemorrhage in last 21 days

Puncture of inaccessible artery within 7 days

Coagulation or Platelet abnormalities
Thrombolysis Protocol








t-PA (Alteplase) – given 1 time over 1 hr IV
Manage Blood Pressure aggressively postThrombolytic  Keep SBP < 185 & DBP < 105
Observe in ICU for first 24 hours
No other antithrombotic agents/ aspirin/
anticoagulants should be given for next 24 hours
(many ppl have this doubt with unstable PE
management also – no heparin for 24 hrs!)
CT Head at 72 hours
Stroke – Other Rx

Start Aspirin 325 mg daily after an ischemic stroke/ TIA.

Heparin/ LMWH has no role in treatment and has increased bleeding risk
( should be used only for DVT prophylaxis not for treatment of CVA)

Later Start aggrenox for secondary prevention of stroke

Warfarin has no additional advantage over Aspirin – do not use unless
indicated.

Remember complications in acute stroke like SIADH, Pneumonia, UTI and
treat adequately

DVT prophylaxis – all cases ( in hemorrhagic stroke ?? may start within 48
hours)
BP Control

Don’t lower Blood Pressure too low in acute CVA  Lower Blood Pressure
leads to lower perfusion in ischemic CVA
Absolute indications for BP intervention in acute Ischemic CVA

Blood Pressure >220/120 or MAP>140

Target organ dysfunction (Hypertensive Emergencies)  Acute Myocardial
Infarction, Hypertensive encephalopathy, Renal Failure, Aortic Dissection
or Retinal hemorrhage / papilledema.

In haemorrhagic stroke, drop SBP by 25-30% ( preferably keep it around 160
range)  put an A-line , use labetalol/ nitroprusside drips if needed

Stroke Prevention – Antiplatelet
therapy comparision






Treat all patients with ischemic stroke or TIA
with aspirin, 50 to 325 mg/d.
Recognize that aspirin, 25 mg, plus extendedrelease dipyridamole, 200 mg bid, both given
bid, is superior to aspirin alone and should be
considered in all patients with TIA or stroke,
although it is more expensive.
Consider clopidogrel alone, 75 mg/d, in patients
who cannot tolerate aspirin, as it has similar
efficacy to aspirin alone.
Stroke – Other Rx/ Secondary Prevention








Evaluate for carotid stenosis ( carotid u/s)
Evaluate for atrial fibrillation (ekg)
Evaluate for structural heart disease etc
( 2D echo)
Rx if Hyperlipidemia is present ( Ischemic
thrombotic CVA is CAD equivalent).
Tobacco cessation
Carotid Stenosis

Evaluation
 Carotid Artery Duplex Ultrasonography




Standard diagnostic tool for carotid stenosis
Less expensive than MRA
Accuracy for diagnosing severe carotid stenosis




Test Sensitivity: 86% , Test Specificity: 87%

Carotid Magnetic Resonance Angiography
(MRA)



Better than ultrasound at defining carotid anatomy
Accuracy for diagnosing severe carotid stenosis


Test Sensitivity: 95% , Test Specificity: 90%
Symptomatic Carotid Stenosis -Mx
Remember that Endarterectomy carries risk of
significant morbidity





Cognitive changes can occur and there can be a 7% risk of
CVA within 30 days of procedure

Symptomatic patient with carotid stenosis >70%




These patients will significantly benefit from carotid
endarterectomy, even if they are over age 75 years

Symptomatic patient with carotid stenosis 50 to 69%




Benefit from carotid endarterectomy present, even if they are
age > 75yrs. However, here consider it if pt is likely to live >
5yrs

Symptomatic patient with carotid stenosis <50%





No benefit from carotid endarterectomy – do not recommend!
Use all other secondary prevention measures
Asymptomatic Carotid Stenosis > 60% Management

Medical therapy: 5 year risk of CVA 12%
 Hypertension control
 Hyperlipidemia control with Statins
 Clopidogrel (Plavix)
Surgical Procedures: 5 year risk of CVA 6%
 Carotid endarterectomy or
 Angioplasty & carotid stenting ( stenting
can be considered in In symptomatic, highrisk candidates for carotid endarterectomy
& In patients with surgically
unapproachable stenosis)
Hemorrhagic Stroke
Intracerebral

hemorrhage
Intracerebellar Hemorrhage
Sub Arachnoid Hemorrhage
Intracerebral Hemorrhage (ICH)





In any patient with stroke symptoms, obtain NON-CONTRAST CT head
first to r/o hemorrhagic stroke
Most common cause is HTN. Other causes are amyloid angiopathy,
vascular malformations, coagulopathy and cocaine abuse
Clues for Amyloid angiopathy – primarily a lobar hemorrhage and seen
mostly in elderly.
Obtain cerebral angogram to rule out vascular malformations in :







Patients with ICH who are age under 45
Patients who developed ICH after cocaine use

Rx  control BP to maintain SBP between 140 and 160 mm hg ( use IV
labetalol or nitroprusside or nicardipine)
 Mannitol and Hyperventilation to reduce intra cranial pressure
In intracerebellar hemorrhages, realize that its very close to brain stem –
so a mass effect can lead to Brain stem herniation. So, in intracerebellar
hemorrhage, if extensive, will need URGENT SURGICAL
DECOMPRESSION – contact neurosurgeon STAT!







Subarachnoid Hemorrhage

Common causes : Berry aneurysms, AV malformations, Neoplasms
C/F: Severe headache, photophobia, loss of consciousness, papilledema
Prevention :
 Obtain MRA to screen for Berry aneurysms in patients with ADPKD
only if family member is diagnosed with an Intra Cranial Aneurysm or
SAH, if the patient refers to symptoms related to an ICA or a patient
has a high risk job
 In patients with incidental aneurysms, if size > 10 mm  surgery. If
size < 10mm, follow up MRI in 1 to 2yrs.
Diagnosis : Non contrast CT first. If CT –VE, LP if suspicion is high  look
for xanthochromia ( 10% SAH are missed by CT Scans)
RX
 If there is a ruptured aneurysm, rx with surgical clipping in 48 hours
 Nimodipine to prevent post subarachnoid hemorrhage vasospasm and
consequent, iscemic stroke ( Vasospasm after SAH is the most
dreaded complication of SAH  leads to stroke)
 Triple “H” therapy ( Induced Hypertension, hemodilution and
hypervolemia) is also widely used in preventing and treating cerebral
vasospasm after aneurysmal SAH.
Meningitis
Meningitis






Symptoms : Fever, Photophobia,
Headache, Neck stiffness, vomiting,
seizures
Use physical exam findings to confirm a
diagnosis of meningitis.
Look for:






Fever
Nuchal rigidity
Brudzinski's sign
Kernig's sign
Signs of encephalitis, such as weakness and change
in mental status
Meningitis
 Do lumbar puncture to obtain CSF for:
 Protein, glucose, and cell count determinations
 Gram stain and bacterial culture
 PCR testing for enterovirus and HSV if bacterial Gram stain and culture
results are negative and cell counts suggest viral meningitis
 Obtain CT scan before lumbar puncture in patients with: ( HIPFAN)
 Immunucompromised state (I)
 History of CNS disease (H)
 New onset seizures (N)
 Papilledema (P)
 Altered level of consciousness ( suggests encephalitis)(A)
 Focal neurologic signs (F)
Be aware that delay in initiating appropriate antibiotics while awaiting
results of CT scan in patients with bacterial meningitis may result in an
adverse clinical outcome.
Meningitis
Meningitis – Empiric Rx
 Base empiric antibiotic therapy on:
 Patient's age
 CSF gram-stain result
 Potential bacterial pathogens
 Knowledge of local resistance patterns for those
pathogens
 Thereafter, base targeted antibiotic therapy on
culture results and susceptibility data.
 Administer dexamethasone 15 to 20 minutes
before the first antimicrobial dose in adult
patients with suspected meningitis.
Meningitis – Emperical therapy
Predisposing Factor
AGE
<1 month

1 - 23 months

2- 50 years

>50 years

Common Bacterial
Pathogens

Antimicrobial Rx

Streptococcus
agalactiae, Escherichia
coli, Listeria
monocytogenes,
Klebsiella species
Streptococcus
pneumoniae , Neisseria
meningitidis, S.
agalactiae,
Haemophilus
influenzae, E. coli
N . meningitidis, S.
pneumoniae

Ampicillin plus
cefotaxime or ampicillin
plus an aminoglycoside

S. pneumoniae, N.
meningitidis, L.
monocytogenes ,
aerobic gram-negative
bacilli

Vancomycin plus a
third-generation
cephalosporin

Vancomycin plus a
third-generation
cephalosporin
Vancomycin plus
ampicillin plus a thirdgeneration
cephalosporin
Meningitis – Emperical therapy

Predisposing Factor

Common Bacterial
Pathogens

Antimicrobial Rx

  Basilar skull fracture

S. pneumoniae, H.
influenzae, group A
-hemolytic streptococci

Vancomycin plus a thirdgeneration cephalosporin

Penetrating trauma

Staphylococcus aureus,
coagulase-negative
staphylococci (especially
Staphylococcus
epidermidis), aerobic gramnegative bacilli (including
Pseudomonas aeruginosa )
Aerobic gram-negative
bacilli (including P.
aeruginosa ), S . aureus ,
coagulase-negative
staphylococci (especially S.
epidermidis)
Coagulase-negative
staphylococci (especially S.
epidermidis), S. aureus,
aerobic gram-negative
bacilli (including P.
aeruginosa ),

Vancomycin plus cefepime,
vancomycin plus
ceftazidime, or vancomycin
plus meropenem  YOU
ARE Adding an
antipseudomonal antibiotic.

HEAD TRAUMA

Postneurosurgery

CSF shunt

Vancomycin plus cefepime,
vancomycin plus
ceftazidime, or vancomycin
plus meropenem
Vancomycin plus cefepime,c
vancomycin plus
ceftazidime,c or vancomycin
plus meropenem
Fall Prevention - Elderly
FALLS IN ELDERLY

Epidemiology
 Falls occur in >30% of age over 65 years in community
 Serious injury occurs in >20% of falls in older adults
 Most falls occur in and around the patient's home
Risk Factors for falls
 Environmental hazards (most common)
 Altered gait or balance
 Lower extremity Muscle Weakness
 Dizziness or Vertigo
 Syncope
 Postural Hypotension
 Decreased visual acquity
 Arthritis
 Dementia or Altered Level of Consciousness
 Major Depression
 Medication use (especially more than 4 medications)


Class IA Antiarrhythmics, Digoxin, Diuretics , Anticonvulsants ,
Psychotropic medications like Benzodiazepines & Antipsychotics
Screening and Evaluation for fall risk



Get Up and Go Test
Cardiovascular exam




Postural Hypotension , Arrhythmias , Carotid Bruits

Neurologic Exam

Assess coordination and balance , Lower extremity muscle strength
& Proprioception and vibration sense




Miscellaneous exam




Visual Acuity , Joint exam

Diagnostics in cases of fall history








Complete Blood Count
Thyroid Function TeSts
Chemistry panel including Renal Function tests
Serum Vitamin B12
Electrocardiogram
Echocardiogram
Brain Imaging
Get up and Go Test
Technique: Direct patient to do the
following
 Rise from sitting position
 Walk 10 feet
 Turn around
 Return to chair and sit down
Interpretation
 Patient takes <20 seconds to complete test




Adequate for independent transfers and mobility

Patient requires >30 seconds to complete test


Suggests higher dependence and risk of of falls
Fall Risk - Prevention

Use Assistive Devices
 Wear flat, rubber soled shoes
 Use ambulatory aid as needed (cane or
walker)
 Consider Hip protection device




Two convex shields worn inside underwear
pocket
Greatly reduces Hip Fracture Incidences

Wearing pads: 0.39 Hip Fractures per 100 falls
 Not wearing pads: 2.43 Hip Fractures per 100 falls




May reduce Incidence of pelvic fracture
Fall Risk Prevention
Education

Proper lifting technique  No stooping; bend knees
and keep back straight
Optimize Comorbid Conditions

Assess number/type of medications

Check Visual Acuity
Vision <20/60 is a risk for falls
Check for Cataracts
Assess for depth perception





Control systolic Hypertension



Systolic Hypertension affects balance and fall risk



Avoid medications that increase fall risk



Medications causing hypotension or Dizziness
Medications causing Sedation





Benzodiazepines




High risk of falls and Hip Fracture
Highest risk within first 2 weeks of starting
Fall Risk Prevention

Modify home environment
 Consider occupational therapy evaluation
 Hand grips and safety mat in shower
 Treads and handrails in stairway
 Anchor rugs, non-skid rubber mats
 Remove clutter, exposed wire, or cord
 Keep halls and stairways well lit
 Use nightlights in bathrooms and bedrooms
Participate in regular Exercise (30
minutes, 4-5/week)
 Walking Program
 Exercise classes twice weekly reduces fall risk
Headaches
Migraines
Tension headaches
Cluster Headaches
Temporal arteritis
Chronic daily headache
Analgesic Rebound Headache
Post Traumatic Headache
Menstrual Migraine
Sinusitis
Migraines
Migraines - Features

Presence of nausea or vomiting
 Duration of typical untreated headache between 4 and
72 hours
 Pulsatile or throbbing character
 Unilateral location of head pain
 Level of disability associated with typical attacks and
impairment of usual daily activities ( any day during past
3 months)
 Photophobia and phonophobia
 Presence of at least 4 of the symptoms above has
sensitivity of 29% for detecting migraine and specificity
100%
 Presence of at least 3 of the symptoms above has
sensitivity 80 and specificity 94

Migraine - Features
 Consider using the pre-test probability of migraine and
the number of symptoms in the POUND mnemonic to
calculate the post-test probability of migraine.
 In applying the mnemonic, determine whether the
headache is:
 P ulsatile in quality
 Approximately O ne-day's duration (between 4 and 72
hours)
 U nilateral in location
 Accompanied by N ausea or vomiting
 Of D isabling intensity
Post-test Probabilities of Migraine Based on the
Number of Migraine Features on History Compared to
a Reference Standard of Headache Expert Neurologist
Clinical Evaluation
Result

Pretest Probability (%)†
20

50

80

5

15

50

3 migraine symptoms*

75

94

99

4 or 5 migraine symptoms*

90

96

>99

0, 1, or 2 migraine symptoms*
Migraine - Diagnosis






No need for neuroimaging for migraine patients
and a normal neurologic examination
Obtain imaging only if atypical headache
features ( worst headache, headache awakening
from sleep, headache
Obtain an ESR in patients over age 50 who have
new-onset headache to exclude the possibility of
temporal arteritis.  Remember, however, that
an elevated ESR (>30 mm/h) is not diagnostic of
temporal arteritis and that a temporal artery
biopsy is needed for diagnosis.
Migraines - Rx




Identify and avoid dietary “triggers,” including:
Caffeine withdrawal , Nitrates and nitrites in
preserved meats, Phenylethylamines in aged
cheeses, red wines, beer, champagne,
chocolate, and monosodium glutamate in Asian
and other prepared foods , Dairy products ,Fatty
foods
Recommend specific behavioral therapies for
migraine patients, including relaxation training,
biofeedback, or cognitive-behavioral therapy
Migraines – Drug Rx
Acute Migraine Rx
Drug Rx






Use migraine-specific agents (e.g., triptans,
dihydroergotamine, ergotamine)
- in patients with severe migraine
- in those whose headaches have
responded poorly to NSAIDs or combination
analgesics such as aspirin plus acetaminophen
plus caffeine.
Triptans first choice because they are more
effective and cause less nausea.
All of these agents are contraindicated in the
presence of CAD. (Ergots contraindicated,
NSAIDS better avoided)
Preventive rx

Indications for daily preventive drug treatment in patients with significant
disability related to frequent or severe migraine attacks (usually at least
2 per month):
 Recurrent headaches that interfere with daily routine
 Contraindication to acute (abortive) therapy , Failure or overuse of
acute therapy , Adverse effects from acute therapy , A preference for
preventive therapy
 Choose among the following effective classes of agents, listed in order
of strength of evidence for efficacy:
 Non-selective, β-antagonists
 Anticonvulsants : VALPROIC ACID, TOPIRAMATE  Be aware that
valproate is the only antiepileptic drug approved by the FDA for
migraine
 Antidepressants, calcium antagonists, ARB ( Candesartan)
 Consider perimenstrual preventive treatment with a triptan for
menstrually associated migraines. ( also ocpills without off period,
Seasonale etc).
 Try a drug therapy for at least 2 months before changing the agent.

Consider tapering or discontinuing preventive treatment after a
sustained reduction in headache frequency that lasts 6 to 12 months.
Referral

Refer the following to a migraine specialist:
 Intractable migraine (status migrainosus)
 Medication-overuse headache or chronic
migraine (sometimes described as
“rebound” headache, transformed
migraine, or chronic daily headache)
 Analgesic dependency (especially narcotic
analgesics)
Cluster Headaches
Cluster Headaches









Excruciating headache pain
Duration of typical attack - 15 to 180 minutes
Symptom-free intervals vary from 1 to 48 hours
Frequency : up to 8 attacks/day
Headache attacks come in clusters daily or at variable
periods.
Nausea, photophobia and phonophobia
Tends to occur more in nights- The headaches have a predilection for

the first rapid eye movement (REM) sleep phase so the cluster patient will awaken
with a severe headache 60 to 90 minutes after falling asleep

Look for presence of autonomic symptoms : Tearing,
Conjunctival injection, Ptosis , Rhinorrhea , Nasal
congestion , Forehead and facial sweating and Miosis
 Note the behavior during attacks (usually agitated, unlike
in migraine where the patient is quiet and withdrawn!)
----------edited ---
Cluster Headache - Physical

During an attack, look for autonomic features
that will help you diagnose this headache:
 Ipsilateral ptosis
 Miosis
 Eyelid edema
 Lacrimation
 Rhinorrhea
 Nasal congestion
 Forehead and facial sweating
 Conjunctival injection
ANY OF ABOVE CLUES AT THE TIME OF
HEADACHE CAN HELP IN DIAGNOSIS

Investigations
Cluster headache is a clinical diagnosis.
 Obtain neuroimaging with MRI and MRA in
patients with:
 Atypical headache features OR Abnormalities
on neurologic examination OR Lack of response
to previous treatments
  Order an ESR to look for temporal arteritis in
patients over age 50 with headache and visual
loss.
 Obtain a polysomnogram in patients with
headache to identify sleep-disordered breathing
(sleep apnea), especially those with snoring or
daytime sleepiness.

Treatment

Identify and recommend that patients avoid trigger
factors of cluster headache such as: Alcohol , Smoking ,
Solvents , Altitudes above 5000 feet , High temperatures
, Altered sleep schedules, e.g., afternoon naps, work
shift changes, traveling across time zones, delay or
advanced sleep phase
 Acute Rx : Give 100% oxygen administered by
nonrebreather face mask at 7 to 15 L/min for 15 to 30
minutes and repeat as needed.
 To treat acute attacks, consider using separately and
then combine as necessary:
 Subcutaneous or intranasal sumatriptan, or nasal
zolmitriptan
 Intramuscular, subcutaneous, or intravenous DHE
 Oral or rectal ergotamine
 Other options : oral corticosteroids, Ipsilateral occipital
nerve block

Trigeminal Neuralgia
Clinical features








Pain lasts from few seconds to minutes.
“Electric” like in character. Can be spontaneous
or triggered by light touch, talking or eating.
Distributed trigeminally (usually second or third
divisions), either alone or in combination
First division pain around the eye or forehead
occurs in 10%-20% of patients, often with pain in
other parts of the face, usually mid-cheek and
upper lip or teeth
Usually unilateral
Pain is relieved by carbamazepine or
oxcarbazepine but not narcotics or milder
analgesics  Consider using carbamazepine
administration as a diagnostic maneuver
Clinical Features




Refractory period of pain after stimulation
of the trigger area (cannot elicit pain again
by touching or pushing immediately after a
painful attack)  Highly predictive of
trigeminal neuralgia.
Neurological exam is usually normal 
Note that abnormal neurological findings
are indicative of diseases other than
trigeminal neuralgia.
Treatment


Avoid PAIN triggers  Stay away from the
direct blast of an air conditioner , Cover
the face before going out into a cold wind,
using covers that do not touch the face or
that are not tight fitting , Avoid foods that
trigger the pain for an individual patient
(e.g., hot or cold drinks, foods that require
much chewing)
Treatment








Mild, infrequent pain  no treatment
Drug of choice for Trigeminal neuralgia 
Oxcarbazepine or Carbamazepine
If pain persists, add another drug, such as
gabapentin, baclofen, lamotrigine, or pregabalin
If pain continues despite drug therapy  needs
neurosurgical intervention  gamma knife
radiosurgery or percutaneous balloon
microcompression of trigeminal division that is
causing symptoms.
Tension Headache
Tension Headaches









Most common form of headache
Last from 30 minutes to 7 days
Typically have bilateral location
Have a nonpulsating pressing or tightening quality
described by patients as a “band-like” constriction
around their head,
Are mild to moderate in intensity
Do not prohibit activity. ( Unlike migraines)
There is no aggravation of headache by using stairs or
by doing any similar routine activity; not associated with
nausea or vomiting (although anorexia may occur);
neither associated with both photophobia and
phonophobia (but may exhibit one or the other)
Treatment










Try Acetaminophen first
NSAIDS if no response to tylenol. OTC NSAIDS and then
prescription NSAIDS
Combination Medications: Aspirin or acetaminophen (or both of
these analgesics) are often combined with caffeine or a sedative
drug in a single medication. For example, Excedrin combines
aspirin, acetaminophen and caffeine  Combination drugs such as
this may be more effective than are pure analgesics for pain relief.
Many combination drugs are available over-the-counter, analgesicsedative combinations can be obtained only by prescription because
they may be addictive and can lead to chronic daily headache.
Advise pts not to use these drugs more than two days a week, as
they can lead to chronic daily headache
Opiates, or narcotics, are rarely used because of their side effects
and potential for dependency. These include codeine combined with
acetaminophen
Chronic Daily Headache
Rebound Headaches
Analgesic overuse headaches
Chronic Migraine
CDH






Chronic daily headache" (CDH) includes a
variety of headache types, of which
chronic migraine is the most common.
Medication overuse, or drug rebound
headache, is the most treatable cause of
refractory daily headache.
A pathologic underlying cause should be
considered in patients with recent-onset
daily headache, a change from a previous
headache pattern, or associated
neurologic or systemic symptoms.
CDH – d/d
Primary Headaches
Headache Duration > 4hrs

Headache Duration < 4hrs

Chronic (transformed) migraine

Strictly unilateral-prominent
autonomic features(SUNCT)

Chronic tension-type headache

Cluster headache

New daily persistent headache

Paroxysmal hemicrania

Hemicrania continua

Trigeminal neuralgia
Cough headache
Benign exertional headache
Headache associated with sexual
activity
CDH – d/d
Secondary Headaches

















Headache associated with vascular disorders
Arteriovenous malformation
Giant cell arteritis
Carotid dissection
Vasculitis
Headache associated with nonvascular intracranial disorders
Neoplasm
Idiopathic intracranial hypertension (pseudotumor cerebri)
Infection
Post-traumatic headache
Subdural hematoma ( chronic)
Myofascial pain
Cervical spine disorders
Temporomandibular joint dysfunction
Headache caused by sleep disorders
Obstructive sleep apnea – early morning headache after sleep









CDH

Most patients with CDH have chronic
(transformed) migraine.
There is a hx of episodic migraine that has
evolved (transformed) over time into a pattern of
almost daily headaches.
These daily headaches may be mild, but
migraine flares may continue to be
superimposed on the daily headache symptoms.
The most common causes of migraine
transformation  a)frequent headaches at
baseline and b) obesity.
Other modifiable risk factors for transformation
include medication overuse, snoring, and
stressful life events.












Drug rebound & Medication
overuseto improve despite use of
Patients who do not stop analgesic overuse fail

preventive therapy  patients who stop taking analgesics on a daily basis have a
marked reduction in frequency of headache
Drug rebound headache is a common treatable cause of transformed migraine.
Patients who have drug rebound headache are refractory to usual acute and
prophylactic interventions  CLUE : The patient who repeatedly presents to the
emergency department requesting narcotics for headache relief most commonly
has drug rebound headache.
The sustained use of these medications more than three days per week is
sufficient to develop drug rebound headache.
All headache medications, including triptans,have the potential to cause drug
rebound headache.
Most common agents that cause drug rebound headache are narcotics, butalbital
products, and combination products containing caffeine.
Be alert to recognize signs of secondary headache in patients who are selfmedicating frequently  Only after a careful evaluation for secondary headache
should drug rebound headache be suspected in patients with medication overuse.
Clinical Approach in Patients with Chronic Daily Headache
1.Treat medication overuse, if present ( next slide)
2.Select pharmacologic therapies
3.Treat potential underlying pathology  Myofascial pain, Physical therapy,
Temporomandibular treatment, Psychiatric comorbidity (antidepressants, anxiolytics)
and Sinus evaluation and treatment etc
4.Limit symptomatic medication use to two days per week (after withdrawal
["detoxification"] is completed).
Recommend use of nonsteroidal anti-inflammatory drugs.
Recommend use of triptans for migraine flares.
Avoid use of medications prone to drug rebound, especially combination analgesics,
caffeine-containing compounds, butalbital products, and narcotics.
5.Consider behavior therapy
Encourage lifestyle management, Regular exercise, Regular meals; no caffeine; migraine
diet
Sleep hygiene
Stress reduction
Biofeedback
Cognitive-behavior therapy
6.Monitor progress (using headache calendar).
Treatment of Medication Overuse (Drug Rebound Headache)

1.Withdrawal of symptomatic medications, including caffeine.
A.Gradually taper medications in patients where physiolgic withdrawal is a
concern (i.e., narcotics, butalbital).
B.Use abrupt withdrawal or taper medications in all other patients.
2.Preventive therapy
Any preventive therapy, or combination medicine for headache
3.Transition therapy

Daily migraine-specific therapy
Dihydroergotamine (DHE), intranasal, intramuscular, or intravenous
Long-acting triptan: naratriptan (Amerge) or frovatriptan (Frova)
B.Anti-inflammatory agents
Short course of corticosteroids
Long-acting nonsteroidal anti-inflammatory drugs
4. Rescue therapy, as needed
A.Non-narcotic analgesics: parenteral ketorolac (Toradol)
B.Antiemetics
C.Sedating antihistamines: diphenhydramine (Benadryl) or hydroxyzine (Atarax)
Benign Intracranial
Hypertension
Pseudotumor Cerebrii
Pseudotumor Cerebri









Female to male ratio = 8:1
Females in reproductive age group – common
Obesity and recent weight gain - an important risk
factor.
Pregnancy can precipitate
Commonest complaint – headache. Can cause vision
loss if papilledema is severe.  there is no altered level
of alertness, cognitive impairment, or focal neurological
findings that are usually associated with the elevated
ICP
Physical : papilledema
Diagnosis : CT head, Lumbar puncture – elevated
opening pressure. Symptoms improve after LP which is
highly suggestive of BIH. Send CSF to all studies to r/o
other etiologies
Treatment
Medical Rx
 Weight loss ( goal 1lb/week for 2 months
or longer)
 Low sodium diet
 Diuretics : Acetalozamide, Furosemide
Surgical Rx:
Optic nerve fenestration
Lumbo-subarachnoid Peritoneal shunting
Bell’s Palsy
Bells Palsy

Because Bell's palsy is a diagnosis of exclusion, make
every effort to exclude other identifiable causes of facial
paralysis
Consider the following:
 Lyme disease : Obtain Lyme disease testing in patients
with acute facial paralysis in endemic areas (Obtain an
ELISA serology for B. burgdorferi in endemic areas if
exposure is likely  Obtain confirmation by Western blot
testing if the ELISA result is positive. )
 Otologic disease ( chronic otitis media, cholesteatoma)
 Neoplasm
 Neurologic diseases, such as stroke and MS

Bells Palsy – Clues in Hx
















Rapidity of onset  Onset over more than 3 weeks strongly suggests neoplastic origin
Hearing loss  Hearing loss is not usually associated with Bell's palsy  Sensorineural
hearing loss suggests Ramsay Hunt syndrome, infectious or neoplastic processes in the
middle ear, cerebella pontine angle affecting the brain stem.  Conductive hearing loss
suggests infectious/neoplastic processes within the temporal bone, especially otitis
media/mastoiditis or cholesteatoma
Periauricular pain  Mild to moderate pain is common for several days before or after
onset of paralysis. Severe persistent pain suggests Ramsay Hunt syndrome or neoplasm
Bilateral facial palsy  Unlikely to be Bell's palsy. Consider Lyme disease, Guillain-Barré,
sarcoidosis, infectious mononucleosis, myasthenia gravis, botulism, acute porphyria,
amyloidosis
Decreased tearing or salivary flow  Common in Bell's palsy secondary to involvement of
parasympathetic fibers that travel with the seventh nerve to the lacrimal and salivary
glands
Dysgeusia  Common in Bell's palsy due to involvement of the chorda tympani
Duration of paralysis  Complete paralysis persisting beyond 6 months. There is always
some recovery from Bell's palsy. Absence of any signs of return of function suggests a
neoplastic etiology
Eye discomfort  A common and important problem in Bell's palsy. It is due to poor
corneal hydration and corneal drying secondary to diminished lacrimation and absent blink
reflex
Decreased visual acuity  A common and important problem in Bell's palsy. It is due to
poor corneal hydration and corneal drying secondary to diminished lacrimation and absent
blink reflex
Bells Palsy – Clues in Physical








A complete ear exam is important to exclude otologic
reasons for seventh nerve paralysis. A maculopapular or
vesicular rash in external ear canal suggests Ramsay
Hunt syndrome
Neurologic exam for segmental focal paralysis  Less
than the entire half of the face is involved; e.g.,
involvement of only the mouth or only the eye 
Neoplasm of parotid most likely etiology, rather than
Bell's palsy, especially if associated with persistent pain
Neurologic exam for forehead-sparing facial paralysis 
Indicates CNS etiology. Consider stroke ( UMN Lesion)
Neurologic exam for balance disturbance  Consider
other neurologic disorder or Ramsay Hunt syndrome
rather than Bell's palsy. Ramsay Hunt syndrome
produces balance disturbance by involvement of the
vestibular nerve and/or semicircular canals
Treatment








Prescribe a short course of high-dose
prednisone (1 mg/kg·d to 70 mg/d for 7 days,
then tapered) for Bell's palsy as soon as the
diagnosis is made, provided that the patient is
seen within 1 week of the onset of paralysis, and
regardless of the degree of paralysis.
Consider antiviral therapy:
Acyclovir, 200 mg orally, 5 times per day
Valacyclovir, 500 to 1000 mg orally, three times
per day
Famciclovir, 500 mg orally, three times per day
Treatment

Manage dry eye aggressively to avoid
ophthalmologic complications  Until complete
eye closure and a good blink reflex return, patch
the eye at night, and ensure that the patch itself
does not touch the cornea and abrade it,
Prescribe artificial tears as needed (as often as
every 15 minutes) during the day, and an ocular
lubricant at night.
 Refer to physical therapist :
 If loss of facial movement is incomplete
 After recovery of movement has begun

Follow-up
Identify patients who experience atypical
recovery or develop treatment side
effects.
 Reevaluate the patient within the first 4 weeks.
 Evaluate the patient about every 3 months
thereafter.
 Instruct the patient to call immediately if any of
the following develop:




Persistent eye pain
Hearing loss or vertigo
New neurologic signs or symptoms
Multiple Sclerosis
Symptoms n Signs














Sensory loss or paresthesias
Partial or complete monocular loss of vision associated with pain (optic
neuritis)
Blurred vision or impaired color vision
Motor weakness suggesting myelitis syndrome, especially paraparesis
Imbalance due to ataxia or sensory ataxia
Spinal or limb paresthesias elicited by neck flexion (Lhermitte's sign)
Bladder or bowel dysfunction such as urgency, incontinence
Diurnal fatigue
Heat sensitivity, causing worsening of fatigue or neurologic symptoms
Memory loss or other cognitive symptoms
Diplopia
Facial pain consistent with trigeminal neuralgia
Dysarthria or dysphagia
M.S Classification
Using the neurologic history, determine if the subtype of MS is:








Relapsing remitting: clinical onset occurs with a “relapse” (“attack”;
“exacerbation”; “flare-up”) of neurologic symptoms or signs that fully
resolve or leave minimal residual deficit
Primary progressive: clinical onset is insidious and worsening of
symptoms occurs gradually over months to years
Secondary progressive: sustained gradual worsening of baseline
status, even between relapses, in patients who initially had
relapsing-remitting disease
Progressive relapsing: a primary progressive onset but the later
establishment of discrete clinical relapses





Obtain:
Brain MRI to assist with MS diagnosis and
to exclude coexisting or other neurologic
disorders
Spinal cord MRI if needed to show
dissemination in space or to exclude a
compressive lesion






Do a lumbar puncture when appropriate based on 2005
Revised McDonald Criteria, usually when an MRI does
not establish dissemination of white matter lesions in
space.
Consider a positive result for purposes of 2005 Revised
McDonald Criteria to be presence of oligoclonal IgG
bands in CSF and not in serum, or an elevated IgG
index.
If MS cannot be confirmed and another diagnosis is not
made, consider serial clinical follow-up examinations and
repeated MRI testing to detect clinical or radiological
changes that may indicate MS.
Revised McDonald Criteria
for Diagnosis of MS
Clinical
(Attacks)

Objective
Lesions

Additional Requirements to Make Diagnosis

2 or more

2 or more

None; additional evidence desirable but must be consistent with MS

2 or more

1

Dissemination in space by MRI or positive CSF and 2 or more MRI lesions
consistent with MS or further clinical attack involving different site

1

2 or more

Dissemination in time by MRI or second clinical attack

1
(monosympto
matic)

1

Dissemination in space by MRI or positive CSF and 2 or more MRI lesions
consistent with MS
AND
Dissemination in time by MRI or second clinical attack

1 (progression
from onset)

1

One year of disease progression (retrospectively or prospectively determined)
AND
Two of the following:
a) Positive brain MRI (nine T2 lesions or four or more T2 lesions with positive
visual evoked potentials)
b) Positive spinal cord MRI (two focal T2 lesions)
c) Positive CSF

Definition of positive MRI—3 out of 4 of the following: 1 gadolinium-enhancing lesion or 9
T2 hyperintense lesions if no gadolinium-enhancing lesion; 1 or more infratentorial
lesions; 1 or more juxtacortical lesions; 3 or more periventricular lesions. Note: A spinal
cord lesion can be considered equivalent to an infratentorial lesion in the brain. Thus, an
enhancing spinal cord lesion is considered to be equivalent to an enhancing brain lesion,
and individual spinal cord lesions can contribute together with individual brain lesions to
reach the required number of T2 lesions.
MS - Admission
Hospitalize patients with:
 Progressive, rapid, or severe deterioration
in bulbar dysfunction (especially
swallowing), ambulatory function, or level
of consciousness
 Medical disorders requiring inpatient
evaluation and therapy, such as infections
or pressure ulcers
Treatment





Use an immunomodulatory therapy (βinterferon preparation or glatiramer
acetate) in patients with RRMS to reduce
rates of clinical relapse and prevent new
MRI lesions.
Natalizumab for patients with established
relapsing MS who respond inadequately
to or cannot tolerate other approved MS
immune therapies to delay worsening of
physical disability and reduce the
frequency of clinical exacerbations.
Treatment













Consider the use of interferon-β for patients with secondary
progressive MS
Initiate corticosteroid therapy such as methylprednisolone, 1000 mg/d
iv for 3 to 5 days, in patients with exacerbations of MS causing
functional impairment
Treat refractory MS-related fatigue with amantadine, 100 to 200 mg/d;
aspirin, 1300 mg/d; or modafinil, 200 mg/d.
Treat appropriately any other medical disorders or clinical situations
that might worsen MS symptoms or signs such as systemic infection, a
worsening or new medical problem, or medication change
Prescribe an antispasticity drug such as baclofen, tizanidine, or
diazepam for patients with muscle spasms or excessive spasticity
interfering with function.
Prescribe tricyclic antidepressants such as amitriptyline or nortriptyline,
with or without analgesics, for patients with constant epicritic (prickling,
burning) neuropathic pain.
Prescribe anticonvulsant medications such as carbamazepine,
oxcarbazepine, or gabapentin for patients with trigeminal neuralgia or
similar paroxysmal neuropathic pain (shooting, jabbing pain)
associated with MS.
Follow-Up







Determine if the patient is experiencing significant and
unacceptable side effects and try to minimize them, for
example:
In patients on corticosteroid therapy, monitor blood pressure
and dietary and blood sugar issues
In patients on interferon therapy,
 Monitor for injection-site reactions, skin necrosis,
depression or suicidal ideation, flu-like symptoms, and
obtain laboratory tests every 6 months:
 CBC with differential
 AST
 ALT
 Alkaline phosphatase
 In patients using any of the β-interferon products, consider
measurement of serum neutralizing antibody status at 12
months and at 24 months after starting therapy or at the
time of a clinical relapse
In patients on immunosuppressive therapy, avoid live vaccines
and unnecessary exposure to viral illnesses
Myasthenia Gravis
Symptoms N Signs

Symptoms
 Ptosis or diplopia – esply, new onset
 Painless difficulty with repetitive tasks
 Painless difficulty in climbing stairs, getting up from a chair, or
walking
 Painless difficulty holding arms over head
 Dyspnea or difficulty swallowing or chewing
 Difficulty holding the head up, or neck muscle soreness
Signs:
 Ptosis
 Extraocular muscle weakness
 Facial weakness
 Difficulty swallowing
 Effort-related dysarthria  dysarthria that worsens with continued
speech.
 Proximal muscle weakness
 Neck weakness











Diagnosis

Anti-Ach Receptor antibodies are diagnostic.
Anti-AchR antibodies are 50% sensitive and 99 specific
Anti-Musk antibodies are 40% sensitive and 99% specific.
In patients with generalized myasthenia who are
seronegative for anti-AChR antibodies, obtain anti-MuSK
antibodies.
In patients with purely ocular signs of myasthenia who are
negative for anti-Musk and Anti-AchR –ve, diagnose with
pharmacologic and electrophysiologic tests ( Edrophonium
test, EMG)  the sensitivity of EMG is 98% and specicficity
is 50-90%.
Consider this diagnosis in presence of classical clinical
features even in the absence of antibodies.
CT chest to r/o thymoma in a pt with newly diagnosed
myasthenia
R/O common co-existent conditions – Hashimato’s
thyroiditis, DM
Diagnosis




If all tests show negative results  Consider
additional neurophysiologic, serologic,
neuroimaging, CSF, genetic, and muscle
biopsy testing
If no other diagnosis can be made, consider
repeated myasthenia gravis testing in future
Differential Diagnosis








MS  History or physical findings of autonomic, special sensory (visual
acuity or visual fields, hearing), or sensory involvement is classic in MS and
is not seen in myasthenia
ALS  Progressive course, symptoms and signs of both upper and lower
motor neuron disease, asymmetry of weakness early in the disease course,
and fasciculations and lack of involvement of eye movement, sensory or
autonomic, are classic for ALS  Unless there is an independent cause of
diplopia or abnormalities of eye movement, the presence of these is
essentially incompatible with ALS  Distinction from myasthenia is
particularly difficult for bulbar ALS, but the presence of significant tongue
atrophy and/or fasciculations, or the presence of signs of upper motor
neuron dysfunction (brisk jaw jerk) suggest the diagnosis of motor neuron
disease ( ALS)
Hypothyroidism  obtain TFTs
Botulism  Descending progressive Paralysis
Eaton-Lambert Syndrome  50%-60% pts have small cell lung ca.
Repetitive stimulation studies cause improvement and this distinguishes
this disorder from myasthenia gravis
Treatment












Ach esterase inhibitors for symptoms  Pyridostigmine,
Neostigmine
Consider immunotherapy based on the course of an
individual patient  treat with one or more
immunosuppressive or immunomodulatory therapies
including:
Corticosteroids such as prednisone or prednisolone
Azathioprine
Mycophenolate mofetil
Cyclosporine
Cyclophosphamide
Plasma exchange
Intravenous immunoglobulin IN MYASTHENIA CRISIS
Treatment




Consider modifying the extent of different
types of physical activity including: Heavy
physical effort , Prolonged speaking , Diet
(soft rather than regular) & Prolonged
reading
Recommend thymectomy in all patients
with evidence of thymoma on chest CT or
MRI unless extensive local spread has
already occurred.
Drug induced Myasthenia


Penicillmine, Aminoglycosides can cause
drug-induced myasthenia  early
diagnosis and discontinuation of the drug
will prevent worsening of symptoms
ALS
Symptoms











Weakness of gradual onset , Muscle wasting , Muscle twitching
( suggests LMN lesion), Muscle cramps , Clumsiness , Muscle
stiffness ( UMN impairment), Changes in voice or articulation ,
Difficulty swallowing , Difficulty breathing ( bulbar involvement)
Spread of symptoms within one region or to additional regions
defined as cranial, cervical, thoracic, and lumbo-sacral  gradual
progression of symptoms suggesting a degenerative disease
Confirm the absence of symptoms that are not typical for ALS,
including:
Predominant sensory symptoms ( favors demyelinating diseases,
neuropathy)
Pain as predominant symptom ( favors radiculopathy, myelopathy)
Bowel or bladder incontinence ( favors cervical myelopathy,
demyelinating diseases)
Cognitive impairment
Ocular muscle weakness ( favors myasthenia)
Signs
Muscle wasting and weakness
 Fasciculations  triggered by muscle contraction or
tapping on a muscle
 Spasticity  suggests UMN involvement
 Hyperreflexia
 Hyporeflexia
 Brisk jaw jerk/ gag reflex
 Evaluate for respiratory muscle weakness looking for
 Use of accessory muscles
 Paradoxical breathing
 Low vocal volume
 Lack of breath support
 Tachypnea

Diagnosis




EMG : Look for electromyographic
evidence of active and chronic
denervation in at least two of four regions
of the CNS ( Brainstem, cervical spinal
cord, thoracic spinal cord, lumbosacral
spinal cord)
Do neuroimaging to r/o other conditions.
Treatment

Offer noninvasive ventilatory support to patients with ALS
and respiratory insufficiency.
 Monitor respiratory status in ALS patients:






Consider noninvasive ventilatory support:










Do serial FVC or sniff nasal pressure measurements
Ask about symptoms of respiratory insufficiency such as insomnia,
daytime fatigue, morning headaches
Obtain overnight pulse oximetry or polysomnography as needed

In patients with symptoms of respiratory insufficiency
When FVC (sitting or supine) is at or below 50% of predicted normal
If there is evidence of nocturnal hypoventilation (O2 saturation <90%),
even if sitting and supine FVC are not significantly reduced

Consider mechanical insufflation-exsufflation for airway secretion
management in patients with insufficient cough.
Consider a portable suction machine for secretion management.
Physical therapy
Occupational therapy
Speech therapy
Treatment
Prescribe riluzole, 50 mg po bid.
Use caution with riluzole and any of its components in patients who:
 Have abnormal liver function , Have renal insufficiency , Are elderly ,
 Monitor for adverse effects of riluzole treatment, Abdominal pain,
anorexia, diarrhea , Arthralgia, asthenia







Nausea, vomiting
Hypertension, tachycardia (rare)
ALT elevations (rare)
Jaundice (rare)
Neutropenia (rare)
Obtain LFT and blood counts every month for the first 3 months,
then every 3 months for the first year, and periodically thereafter
while the patient is on riluzole.
Neurosyphilis
Refer to ID slides under “Syphilis”
Delirium
Refer to Psychaitry Slides
Dementia
Risk Factors for Dementia













Age
Family history of dementia
History of hypertension
History of head injury
Low education attainment (<10 years)
Alcohol abuse
Current ASA use
Pesticides and fertilizers
Liquid plastics or rubber
? Screen patients  Because the incidence of dementia increases
with age, screening has generally been considered for people over
a certain age, usually 60 or 65, but there is currently no evidence on
which to base such recommendations
Screening test – MMSE – Score of 26 or lower is dementia
Diagnosis

Ask the patient and the family member about:
 Memory loss
 Getting lost
 Word-finding difficulties
 Impaired ADL, such as dressing, grooming, and
housework.
 Changes in:
 Personality , Mood m Energy , Appetite , Sleep ,
Enjoyment of activities ( Depression is an important
differential diagnosis)
 Behavioral changes, including:
 General activity level , Eating , Drinking , Sleep , Sex
 Neurologic deficits, including:
 Gait abnormalities
 Falls
 Weakness
 Clumsiness
 Sensory abnormalities
 Abnormal movements
 Incontinence
Diagnosis




Nature and time course of cognitive problems, especially acute vs.
subacute course, or evidence of fluctuating level of consciousness.
The sequence in which the cognitive difficulties and other symptoms
developed should be chronicled



An acute change suggests delirium or a recent CNS event



Fluctuating level of consciousness suggests delirium.







Temporal association with a change in medication suggests a
causal relationship.
Classic Alzheimer disease presents with early loss of short-term
memory, language, and visuospatial abilities, but preserved
personality and normal neurologic exam
Personality change  May suggest frontal lobe pathology (e.g,
stroke, tumor, or frontotemporal dementia)
Physical exam
Do a mental status exam to evaluate:
 Level of alertness
 Short- and long-term memory
 Orientation
 Concentration
 Abstract reasoning
 Language (naming, vocabulary, fluency, repetition, comprehension)
 Visuospatial abilities (clock drawing, design copy)
 Cortical-sensory integrative function (neglect, left-right
differentiation, stereognosis, and graphestesia)
 Praxis
 Mood
 Hallucinations or delusions
 Apathy
 Do a comprehensive neurologic exam, including cranial nerve,
motor, sensory, reflex, and cerebellar function, to look for concurrent
CNS disease.
D/D = Dementia



















Delirium
Drugs ( esply BZDs)
Depression
Alzheimers
Vascular dementia ( step wise deterioration)
Lewy body dementia ( Parkinsons disease + visual hallucinations)
NPH
Sub dural hematoma
Fronto temporal dementia – including picks disease ( personality
changes)
Vitamin b12 deficiency
Traumatic brain injury
Thyroid disease
Brain tumor
HIV dementia
Huntington disease
Parkinson disease
Chronic alcohol use
CJD
Vascular dementia









Stepwise” deterioration.
Loss of function should be correlated temporally
with cerebrovascular events.
Level of consciousness must be normal to make
the diagnosis.
May also be present in patients with “silent”
strokes, multiple small strokes, or severe diffuse
CVD.
Should be suspected in any patient with
cerebrovascular risk factors, even if a neurologic
exam doesn’t suggest a stroke
CJD









Rapid progression
Early age of onset
Prominent myoclonus
Characteristic EEG pattern of triphasic
sharp waves (1-2 Hz)
Diffusion-weighted MRI may be more
sensitive and specific for the diagnosis of
this condition
Poor prognosis
HIV Dementia





Seen with advanced HIV
Memory disturbance usually accompanied
by lethargy and social withdrawal, as well
as by motor dysfunction (ataxia,
weakness, and incoordination).
Aphasia, apraxia, and agnosia are rare
NPH








Clinical triad of dementia, gait abnormality (slow,
broad-based, impaired turning), and urinary
incontinence.
Dementia is often associated with psychomotor
slowing and apathy.
Dementia and apathy may be the earliest
symptoms
CT scan of head is useful.
If suspicion is high, lumbar puncture with pregait and post-gait monitoring is done.
Ventriculo-peritoneal shunting can be curative in
some patients
Delirium vs. Dementia












Altered level of alertness and attention, often in
conjunction with globally impaired cognition.
Onset may be abrupt, and fluctuating level of alertness is
common.
Older patients often appear to have psychomotor
retardation, and may show the full range of mental status
abnormalities, including depressed or elevated mood,
hallucinations, delusions, and agitated behavior
Delirium must be excluded in order to diagnose
dementia.
Making the diagnosis is critical, because it often reflects
a serious systemic disturbance.
Metabolic derangement, medication effects, and
infection are the most common causes
Alzheimer’s Disease










Gradual memory loss, preservation of level of consciousness,
inefficiency or impairment in ADL performance may also be present.
May initially become manifest when the patient has lost a significant
source of assistance, for example the loss of a spouse or a
significant change in routine (e.g., moving).
Neurologic signs, such as falls, tremor, weakness, or reflex
abnormalities, are not typical early in the disease course.
As the illness progresses, other cortical deficits, such as aphasia,
apraxia, agnosia, inattention, and left-right confusion will develop.
Seizures are present frequently in advanced disease; their presence
earlier in the course suggests a diagnosis other than Alzheimer
disease.
The presenting symptom to the physician may not be a cognitive
problem.
Often, the earliest presenting symptoms are paranoid delusions or
depression, which upon further investigation turn out to be part of a
dementia
Alzheimers - diagnosis












Criteria for clinical diagnosis of PROBABLE Alzheimer's disease
include:
Dementia established by clinical examination and documented by
the Mini-Mental Test, Blessed Dementia Scale, or some similar
examination, and confirmed by neuropsychological tests:
Deficits in two or more areas of cognition
Progressive worsening of memory and other cognitive functions
No disturbance of consciousness
Onset between ages 40 and 90, most often after age 65, and
absence of systemic disorders or other brain diseases that in and of
themselves could account for the progressive deficits in memory
and cognition
Drug therapy - Dementia











Use acetylcholinesterase inhibitors to delay cognitive
decline in Alzheimer disease, dementia with Lewy
bodies, and mixed Alzheimer disease and vascular
dementia
Use donepezil, rivastigmine, or galantamine to delay
progression of symptoms in Alzheimer disease,
dementia with Lewy bodies, and mixed Alzheimer
disease and vascular dementia.
Initiate treatment once the diagnosis has been made and
the patient is medically and psychiatrically stable.
Ensure that treatment is continuous and without lengthy
interruptions.
Increase medication doses monthly until target doses
are reached.
Drug therapy - Dementia







Do not prescribe high-dose vitamin E routinely to slow
the progression of symptoms in Alzheimer disease.
Use memantine to delay cognitive decline in moderate to
advanced Alzheimer disease and vascular dementia. ,
Begin with 5 mg/d.  Increase medication doses weekly
by 5 mg/d until the target dose of 10 mg twice daily is
reached.  Add memantine in patients on a stable dose
of a cholinesterase inhibitor, but do not use it as a
substitute
Consider using Ginkgo biloba extract, 120 to 240 mg/d,
in patients with mild to severe Alzheimer disease, but
recognize that there are insufficient data to recommend
its use.
Replace vitamin B12 in patients with evidence of tissue
deficiency of vitamin B12 (elevated methylmalonic acid
and homocysteine).
Drug Therapy
Treat psychotic symptoms or behavioral disturbances complicating
dementia with drugs

Consider using one of the following antipsychotic medications in the
treatment of psychotic symptoms (hallucinations and delusions) or
behavioral disturbances (aggression, severe irritability, agitation,
explosiveness) if there is a risk of harm to the patient or others, or if patient
distress is significant and non-drug treatments have been ineffective:
Olanzapine:, Risperidone: Quetiapine, Acetylcholinesterase inhibitors
(donepezil, galantamine, or rivastigmine)
 Minimize use of antipsychotics in patients with dementia to the extent
possible by:

Using the lowest effective dose

Restricting use of antipsychotics to patients with hallucinations, delusions,
or agitation, the symptoms for which these medications have proven
efficacy

Treating these symptoms only if they are causing significant problems, such
as distressing the patient or caregiver, jeopardizing a living arrangement, or
necessitating psychiatric hospitalization

Reevaluating the need for continued antipsychotic use on a regular basis

Consider consultation with a geriatric psychiatrist in patients with difficult to
treat symptoms or the development of polypharmacy

Drug therapy - Pseudodementia









Treat patients with significant symptoms of depression
with antidepressant drugs: Sertraline, Paroxetine,
citalopram, fluoxetine, venlafaxine
Avoid agents with prominent anticholinergic effects, such
as amitriptyline and imipramine, in patients with
dementia
Maintain treatment at therapeutic doses for at least 6 to
8 weeks before declaring the trial a failure.
Consider referral to a geriatric psychiatrist if one or two
trials of antidepressants (at therapeutic doses given for
at least 6 to 8 weeks) have failed or cannot be tolerated
due to side effects.
Be aware that all of these drugs can cause or
exacerbate delirium.
Driving - Dementia


Address the patient's driving ability with the patient and caregiver.









Inquire about motor vehicle accidents or near accidents, and changes in
driving habits or patterns.
Advise patients who already show driving impairment that for their own
safety and the safety of others they must no longer drive.

Advise patients who have received the diagnosis of dementia, but
have not yet shown any difficulties with driving, to undergo a driving
evaluation and to refrain from driving before completion of the
evaluation.
Note that driving evaluations are usually available at the local motor
vehicle agency or hospital departments of occupational therapy.
Follow state law with regard to informing the motor vehicle agency
of a patient's impaired driving ability.
For patients who are able to continue to drive, repeat driving
assessment every 6 months.
END
Questions?
Seizure disorders
Parkinson disease
Narcolepsy and cataplexy
insomnia
Guilliane-barre syndrome
Peripheral neuropathies
Carpal tunnel syndrome
Movement Disorders





Essential Tremor
Huntigtons
Parkinsonism
Myoclonus

Más contenido relacionado

La actualidad más candente

Atrial fibrillation management
Atrial fibrillation managementAtrial fibrillation management
Atrial fibrillation management
Basem Enany
 
management of acute coronary syndrome
management of acute coronary syndromemanagement of acute coronary syndrome
management of acute coronary syndrome
Basem Enany
 

La actualidad más candente (20)

Samir rafla ecg arrhythmia for medical students- added amr kamal
Samir rafla  ecg arrhythmia for medical students- added amr kamalSamir rafla  ecg arrhythmia for medical students- added amr kamal
Samir rafla ecg arrhythmia for medical students- added amr kamal
 
Atrial fibrillation...rx
Atrial fibrillation...rxAtrial fibrillation...rx
Atrial fibrillation...rx
 
Atrial Fibrillation 2016
Atrial Fibrillation 2016Atrial Fibrillation 2016
Atrial Fibrillation 2016
 
Atrial fibrillation management
Atrial fibrillation managementAtrial fibrillation management
Atrial fibrillation management
 
Atrial fibrillation ksaus hs 2019
Atrial fibrillation ksaus hs 2019Atrial fibrillation ksaus hs 2019
Atrial fibrillation ksaus hs 2019
 
Acute ischemic stroke
Acute ischemic strokeAcute ischemic stroke
Acute ischemic stroke
 
Atrial fibrillation
Atrial fibrillationAtrial fibrillation
Atrial fibrillation
 
Esc guideline for atrial fibrillation 2020 [dr pranab]
Esc guideline for atrial fibrillation 2020 [dr pranab]Esc guideline for atrial fibrillation 2020 [dr pranab]
Esc guideline for atrial fibrillation 2020 [dr pranab]
 
2014 AHA/ACC Atrial Fibrillation Guidelines
2014 AHA/ACC Atrial Fibrillation Guidelines2014 AHA/ACC Atrial Fibrillation Guidelines
2014 AHA/ACC Atrial Fibrillation Guidelines
 
ATRIAL FIBRILLATION 2016
ATRIAL FIBRILLATION 2016ATRIAL FIBRILLATION 2016
ATRIAL FIBRILLATION 2016
 
A fib
A fibA fib
A fib
 
management of acute coronary syndrome
management of acute coronary syndromemanagement of acute coronary syndrome
management of acute coronary syndrome
 
atrial fibrillation
atrial fibrillation atrial fibrillation
atrial fibrillation
 
Atrial Fibrillation - From Diagnosis to Treatment - St Vincent's Birmingham
Atrial Fibrillation - From Diagnosis to Treatment - St Vincent's BirminghamAtrial Fibrillation - From Diagnosis to Treatment - St Vincent's Birmingham
Atrial Fibrillation - From Diagnosis to Treatment - St Vincent's Birmingham
 
Guia AHA/ACC STROKE 2014
Guia AHA/ACC STROKE 2014 Guia AHA/ACC STROKE 2014
Guia AHA/ACC STROKE 2014
 
Management of atrial fibrillation in critically ill patients
Management of atrial fibrillation in critically ill patientsManagement of atrial fibrillation in critically ill patients
Management of atrial fibrillation in critically ill patients
 
Atrial fibrillation -latest
Atrial fibrillation -latestAtrial fibrillation -latest
Atrial fibrillation -latest
 
Management of acute stroke
Management of acute strokeManagement of acute stroke
Management of acute stroke
 
Relative Contraindications for Thrombolysis in Acute Ischemic Stroke
Relative Contraindications for Thrombolysis in Acute Ischemic StrokeRelative Contraindications for Thrombolysis in Acute Ischemic Stroke
Relative Contraindications for Thrombolysis in Acute Ischemic Stroke
 
What is Atrial Fibrillation (afib)?
What is Atrial Fibrillation (afib)?What is Atrial Fibrillation (afib)?
What is Atrial Fibrillation (afib)?
 

Similar a Neurology[1]

Stroke emergency treatment
Stroke emergency treatmentStroke emergency treatment
Stroke emergency treatment
PS Deb
 
Stroke emergency treatment for 26th march 00
Stroke emergency treatment for 26th march 00Stroke emergency treatment for 26th march 00
Stroke emergency treatment for 26th march 00
PS Deb
 
Supra ventricular tachycardia
Supra ventricular tachycardiaSupra ventricular tachycardia
Supra ventricular tachycardia
Tamil Mani
 
Anaesthetic management of Abdominal aortic aneurysms
Anaesthetic management of Abdominal aortic aneurysmsAnaesthetic management of Abdominal aortic aneurysms
Anaesthetic management of Abdominal aortic aneurysms
Abhijit Nair
 

Similar a Neurology[1] (20)

Archer Neurology for USMLE Step 3
Archer Neurology for USMLE Step 3Archer Neurology for USMLE Step 3
Archer Neurology for USMLE Step 3
 
Ischaemic stroke cme
Ischaemic stroke cmeIschaemic stroke cme
Ischaemic stroke cme
 
Acute Coronary Syndromes
Acute Coronary Syndromes Acute Coronary Syndromes
Acute Coronary Syndromes
 
Hypertensive emergencies
Hypertensive emergenciesHypertensive emergencies
Hypertensive emergencies
 
Stroke emergency treatment
Stroke emergency treatmentStroke emergency treatment
Stroke emergency treatment
 
Ischaemic stroke
Ischaemic stroke Ischaemic stroke
Ischaemic stroke
 
Hypertensive emergencies
Hypertensive emergenciesHypertensive emergencies
Hypertensive emergencies
 
Stroke emergency treatment for 26th march 00
Stroke emergency treatment for 26th march 00Stroke emergency treatment for 26th march 00
Stroke emergency treatment for 26th march 00
 
Stroke-and-Spinal-Cord-7-30.ppt
Stroke-and-Spinal-Cord-7-30.pptStroke-and-Spinal-Cord-7-30.ppt
Stroke-and-Spinal-Cord-7-30.ppt
 
Subarachnoid hemorrage –eso guidelines for management
Subarachnoid hemorrage –eso guidelines for managementSubarachnoid hemorrage –eso guidelines for management
Subarachnoid hemorrage –eso guidelines for management
 
Supra ventricular tachycardia
Supra ventricular tachycardiaSupra ventricular tachycardia
Supra ventricular tachycardia
 
Stroke (Cerebrovascular Accident)
Stroke (Cerebrovascular Accident)Stroke (Cerebrovascular Accident)
Stroke (Cerebrovascular Accident)
 
Stroke CVa.pptx
Stroke CVa.pptxStroke CVa.pptx
Stroke CVa.pptx
 
Stroke for under graduates and Post Graduate
Stroke for under graduates and Post GraduateStroke for under graduates and Post Graduate
Stroke for under graduates and Post Graduate
 
Anaesthetic management of Abdominal aortic aneurysms
Anaesthetic management of Abdominal aortic aneurysmsAnaesthetic management of Abdominal aortic aneurysms
Anaesthetic management of Abdominal aortic aneurysms
 
MANAGEMENT OF ATRIAL FIBRILLATION.pptx
MANAGEMENT OF ATRIAL FIBRILLATION.pptxMANAGEMENT OF ATRIAL FIBRILLATION.pptx
MANAGEMENT OF ATRIAL FIBRILLATION.pptx
 
Anaesthesia and ihd
Anaesthesia and ihdAnaesthesia and ihd
Anaesthesia and ihd
 
stroke.pptx
stroke.pptxstroke.pptx
stroke.pptx
 
Acute Coronary Disease
Acute Coronary DiseaseAcute Coronary Disease
Acute Coronary Disease
 
Stroke
StrokeStroke
Stroke
 

Más de S Mukesh Kumar (15)

Pulmonology
PulmonologyPulmonology
Pulmonology
 
Psychiatry ppt
Psychiatry pptPsychiatry ppt
Psychiatry ppt
 
Preventive medicine
Preventive medicinePreventive medicine
Preventive medicine
 
Peds, surg
Peds, surgPeds, surg
Peds, surg
 
Oncology step3
Oncology step3Oncology step3
Oncology step3
 
Ob & gyn
Ob & gynOb & gyn
Ob & gyn
 
Nephrology e tutoring2
Nephrology e tutoring2Nephrology e tutoring2
Nephrology e tutoring2
 
Infectious disease
Infectious diseaseInfectious disease
Infectious disease
 
Hematology
HematologyHematology
Hematology
 
Giant cell chest conference
Giant cell chest conferenceGiant cell chest conference
Giant cell chest conference
 
Gastroenterology
GastroenterologyGastroenterology
Gastroenterology
 
Ethics in medicine
Ethics in medicineEthics in medicine
Ethics in medicine
 
Endocrinology
EndocrinologyEndocrinology
Endocrinology
 
Dermatology
DermatologyDermatology
Dermatology
 
Rheumatology 2
Rheumatology 2Rheumatology 2
Rheumatology 2
 

Último

1029 - Danh muc Sach Giao Khoa 10 . pdf
1029 -  Danh muc Sach Giao Khoa 10 . pdf1029 -  Danh muc Sach Giao Khoa 10 . pdf
1029 - Danh muc Sach Giao Khoa 10 . pdf
QucHHunhnh
 
Jual Obat Aborsi Hongkong ( Asli No.1 ) 085657271886 Obat Penggugur Kandungan...
Jual Obat Aborsi Hongkong ( Asli No.1 ) 085657271886 Obat Penggugur Kandungan...Jual Obat Aborsi Hongkong ( Asli No.1 ) 085657271886 Obat Penggugur Kandungan...
Jual Obat Aborsi Hongkong ( Asli No.1 ) 085657271886 Obat Penggugur Kandungan...
ZurliaSoop
 
The basics of sentences session 3pptx.pptx
The basics of sentences session 3pptx.pptxThe basics of sentences session 3pptx.pptx
The basics of sentences session 3pptx.pptx
heathfieldcps1
 

Último (20)

2024-NATIONAL-LEARNING-CAMP-AND-OTHER.pptx
2024-NATIONAL-LEARNING-CAMP-AND-OTHER.pptx2024-NATIONAL-LEARNING-CAMP-AND-OTHER.pptx
2024-NATIONAL-LEARNING-CAMP-AND-OTHER.pptx
 
ICT role in 21st century education and it's challenges.
ICT role in 21st century education and it's challenges.ICT role in 21st century education and it's challenges.
ICT role in 21st century education and it's challenges.
 
SKILL OF INTRODUCING THE LESSON MICRO SKILLS.pptx
SKILL OF INTRODUCING THE LESSON MICRO SKILLS.pptxSKILL OF INTRODUCING THE LESSON MICRO SKILLS.pptx
SKILL OF INTRODUCING THE LESSON MICRO SKILLS.pptx
 
Understanding Accommodations and Modifications
Understanding  Accommodations and ModificationsUnderstanding  Accommodations and Modifications
Understanding Accommodations and Modifications
 
Sociology 101 Demonstration of Learning Exhibit
Sociology 101 Demonstration of Learning ExhibitSociology 101 Demonstration of Learning Exhibit
Sociology 101 Demonstration of Learning Exhibit
 
1029 - Danh muc Sach Giao Khoa 10 . pdf
1029 -  Danh muc Sach Giao Khoa 10 . pdf1029 -  Danh muc Sach Giao Khoa 10 . pdf
1029 - Danh muc Sach Giao Khoa 10 . pdf
 
Fostering Friendships - Enhancing Social Bonds in the Classroom
Fostering Friendships - Enhancing Social Bonds  in the ClassroomFostering Friendships - Enhancing Social Bonds  in the Classroom
Fostering Friendships - Enhancing Social Bonds in the Classroom
 
Explore beautiful and ugly buildings. Mathematics helps us create beautiful d...
Explore beautiful and ugly buildings. Mathematics helps us create beautiful d...Explore beautiful and ugly buildings. Mathematics helps us create beautiful d...
Explore beautiful and ugly buildings. Mathematics helps us create beautiful d...
 
Unit-IV- Pharma. Marketing Channels.pptx
Unit-IV- Pharma. Marketing Channels.pptxUnit-IV- Pharma. Marketing Channels.pptx
Unit-IV- Pharma. Marketing Channels.pptx
 
General Principles of Intellectual Property: Concepts of Intellectual Proper...
General Principles of Intellectual Property: Concepts of Intellectual  Proper...General Principles of Intellectual Property: Concepts of Intellectual  Proper...
General Principles of Intellectual Property: Concepts of Intellectual Proper...
 
Spatium Project Simulation student brief
Spatium Project Simulation student briefSpatium Project Simulation student brief
Spatium Project Simulation student brief
 
Making communications land - Are they received and understood as intended? we...
Making communications land - Are they received and understood as intended? we...Making communications land - Are they received and understood as intended? we...
Making communications land - Are they received and understood as intended? we...
 
On National Teacher Day, meet the 2024-25 Kenan Fellows
On National Teacher Day, meet the 2024-25 Kenan FellowsOn National Teacher Day, meet the 2024-25 Kenan Fellows
On National Teacher Day, meet the 2024-25 Kenan Fellows
 
ComPTIA Overview | Comptia Security+ Book SY0-701
ComPTIA Overview | Comptia Security+ Book SY0-701ComPTIA Overview | Comptia Security+ Book SY0-701
ComPTIA Overview | Comptia Security+ Book SY0-701
 
SOC 101 Demonstration of Learning Presentation
SOC 101 Demonstration of Learning PresentationSOC 101 Demonstration of Learning Presentation
SOC 101 Demonstration of Learning Presentation
 
Mixin Classes in Odoo 17 How to Extend Models Using Mixin Classes
Mixin Classes in Odoo 17  How to Extend Models Using Mixin ClassesMixin Classes in Odoo 17  How to Extend Models Using Mixin Classes
Mixin Classes in Odoo 17 How to Extend Models Using Mixin Classes
 
ICT Role in 21st Century Education & its Challenges.pptx
ICT Role in 21st Century Education & its Challenges.pptxICT Role in 21st Century Education & its Challenges.pptx
ICT Role in 21st Century Education & its Challenges.pptx
 
Jual Obat Aborsi Hongkong ( Asli No.1 ) 085657271886 Obat Penggugur Kandungan...
Jual Obat Aborsi Hongkong ( Asli No.1 ) 085657271886 Obat Penggugur Kandungan...Jual Obat Aborsi Hongkong ( Asli No.1 ) 085657271886 Obat Penggugur Kandungan...
Jual Obat Aborsi Hongkong ( Asli No.1 ) 085657271886 Obat Penggugur Kandungan...
 
Google Gemini An AI Revolution in Education.pptx
Google Gemini An AI Revolution in Education.pptxGoogle Gemini An AI Revolution in Education.pptx
Google Gemini An AI Revolution in Education.pptx
 
The basics of sentences session 3pptx.pptx
The basics of sentences session 3pptx.pptxThe basics of sentences session 3pptx.pptx
The basics of sentences session 3pptx.pptx
 

Neurology[1]

  • 1. Neurology Archer USMLE Step3 Reviews www.CcsWorkshop.com
  • 2.            Risk Factors - Stroke Age >50 Family history of CAD, CVD, or PVD before age 60 Clinical manifestations of CAD or PVD Hypertension Diabetes mellitus Elevated cholesterol Smoking Hyperhomocysteinemia Carotid bruit ( presene only if luminal stenosis >50%) History of TIA History of paroxysmal or persistent atrial fibrillation
  • 3.     Primary Prevention Of Stroke Control hypertension  Use ACE inhibitor in high-risk patients  if they are hypertensive or diabetic, or if they have already had a vascular ischemic event. Smoking cessation. Achieve glycemic control in diabeteics Start warfarin in most patients with nonvalvular atrial fibrillation in whom sinus rhythm cannot be restored, including (CHADS2)   High-risk patients with previous embolization (TIA/CVA), age > 75, HTN, DM Type II and those with congestive heart failure or decreased left ventricular function Elderly patients to maintain an INR between 1.8 and 2.5
  • 4. Primary Prevention Of Stroke     Do not use warfarin  in patients who cannot have adequate monitoring of INR, cannot receive medications in a predictable fashion, or have increased risk for bleeding (i.e., those with a high risk of traumatic fall (elderly), those with high-risk occupations, or those who have a bleeding disorder). Start antiplatelet therapy in Afib for prevention for storke- ASA 50 to 325 mg po qd, Clopidogrel, 75 mg po qd, or Aggrenox - in low-risk patients with nonvalvular atrial fibrillation (lone afib) - in higher-risk patients who decline anticoagulation, would not comply with INR monitoring, or are at very high risk for bleeding. Discontinue combination estrogen and progestin treatment in healthy postmenopausal women. Consider starting low-dose every-other-day aspirin in otherwise asymptomatic women over the age of 45.
  • 5. Primary Prevention - Stroke  Carotid endarterectomy should be considered in asymptomatic patients with >60% stenosis  who have no other contraindications for surgery (especially severe cardiopulmonary disease), are under age 75, and are expected to live >5 years .
  • 6. Stroke - Management Immediate Neurologic Assessment (<25 minutes)  Alert Stroke Team of possible Thrombolytic. Determine onset of CVA symptoms  Consider Thrombolytics within 3 hours of onset  Physical Examination  Neurologic Examination  Assess Level of Consciousness (Glascow Coma Scale) & Assess Stroke Severity Step 2: Rule-out Hemorrhagic CVA  Imaging    If Head CT suggests intracranial bleeding  Neurosurgery consultation   Reverse anticoagulants or Bleeding Disorder and Manage Hypertension appropriately Head CT negative despite high suspicion for SAH   Obtain urgent noncontrast Head CT (<25 minutes)  Head CT read by radiologist (<45 minutes) Lateral Neck XRay Indications  Altered Level of Consciousness , Trauma Obtain Lumbar Puncture to assess subarachnoid blood  remember, Lumbar Puncture contraindicates Thrombolytics if you are suspecting an ischemic cva also. Head CT negative suggesting Ischemic CVA  Give Thrombolytic Therapy if meets the criteria/ If not, just ASA or Aggrenox.
  • 7. Stroke - Management Step 3: Thrombolytic Therapy (if indicated)  Know Thrombolytic Contraindications  Know indications for Thrombolytic therapy   Persistent neurologic deficits CVA Symptom onset <3 hours prior
  • 8. Stroke - Thrombolytics Inclusion Criteria  Age over 18 years  Clinical Diagnosis of acute Ischemic Stroke  CT Head compatible with Ischemic CVA diagnosis  Known time of onset under 3 hours before Thrombolytics  Do not use intravenous tPA beyond 3 hours of symptoms Exclusion Criteria  Improving or mild neurologic deficit  Seizure at onset  Head Injury or CVA within 90 days  History of Intracranial Hemorrhage  Suspected Subarachnoid Hemorrhage (SAH)  Hemorrhage on CT Head or History suggests SAH even despite negative Head CT (worst headache/ photophobia)  Hypertension refractory to antihypertensives Systolic Blood Pressure over 185 and Diastolic Blood Pressure over 110 ( if elevated, control and then give tpa)  Major surgery in last 14 days  Gastrointestinal hemorrhage in last 21 days  Genitourinary hemorrhage in last 21 days  Puncture of inaccessible artery within 7 days  Coagulation or Platelet abnormalities
  • 9. Thrombolysis Protocol      t-PA (Alteplase) – given 1 time over 1 hr IV Manage Blood Pressure aggressively postThrombolytic  Keep SBP < 185 & DBP < 105 Observe in ICU for first 24 hours No other antithrombotic agents/ aspirin/ anticoagulants should be given for next 24 hours (many ppl have this doubt with unstable PE management also – no heparin for 24 hrs!) CT Head at 72 hours
  • 10. Stroke – Other Rx Start Aspirin 325 mg daily after an ischemic stroke/ TIA.  Heparin/ LMWH has no role in treatment and has increased bleeding risk ( should be used only for DVT prophylaxis not for treatment of CVA)  Later Start aggrenox for secondary prevention of stroke  Warfarin has no additional advantage over Aspirin – do not use unless indicated.  Remember complications in acute stroke like SIADH, Pneumonia, UTI and treat adequately  DVT prophylaxis – all cases ( in hemorrhagic stroke ?? may start within 48 hours) BP Control  Don’t lower Blood Pressure too low in acute CVA  Lower Blood Pressure leads to lower perfusion in ischemic CVA Absolute indications for BP intervention in acute Ischemic CVA  Blood Pressure >220/120 or MAP>140  Target organ dysfunction (Hypertensive Emergencies)  Acute Myocardial Infarction, Hypertensive encephalopathy, Renal Failure, Aortic Dissection or Retinal hemorrhage / papilledema.  In haemorrhagic stroke, drop SBP by 25-30% ( preferably keep it around 160 range)  put an A-line , use labetalol/ nitroprusside drips if needed 
  • 11. Stroke Prevention – Antiplatelet therapy comparision    Treat all patients with ischemic stroke or TIA with aspirin, 50 to 325 mg/d. Recognize that aspirin, 25 mg, plus extendedrelease dipyridamole, 200 mg bid, both given bid, is superior to aspirin alone and should be considered in all patients with TIA or stroke, although it is more expensive. Consider clopidogrel alone, 75 mg/d, in patients who cannot tolerate aspirin, as it has similar efficacy to aspirin alone.
  • 12. Stroke – Other Rx/ Secondary Prevention      Evaluate for carotid stenosis ( carotid u/s) Evaluate for atrial fibrillation (ekg) Evaluate for structural heart disease etc ( 2D echo) Rx if Hyperlipidemia is present ( Ischemic thrombotic CVA is CAD equivalent). Tobacco cessation
  • 13. Carotid Stenosis Evaluation  Carotid Artery Duplex Ultrasonography    Standard diagnostic tool for carotid stenosis Less expensive than MRA Accuracy for diagnosing severe carotid stenosis   Test Sensitivity: 86% , Test Specificity: 87% Carotid Magnetic Resonance Angiography (MRA)   Better than ultrasound at defining carotid anatomy Accuracy for diagnosing severe carotid stenosis  Test Sensitivity: 95% , Test Specificity: 90%
  • 14. Symptomatic Carotid Stenosis -Mx Remember that Endarterectomy carries risk of significant morbidity   Cognitive changes can occur and there can be a 7% risk of CVA within 30 days of procedure Symptomatic patient with carotid stenosis >70%   These patients will significantly benefit from carotid endarterectomy, even if they are over age 75 years Symptomatic patient with carotid stenosis 50 to 69%   Benefit from carotid endarterectomy present, even if they are age > 75yrs. However, here consider it if pt is likely to live > 5yrs Symptomatic patient with carotid stenosis <50%    No benefit from carotid endarterectomy – do not recommend! Use all other secondary prevention measures
  • 15. Asymptomatic Carotid Stenosis > 60% Management Medical therapy: 5 year risk of CVA 12%  Hypertension control  Hyperlipidemia control with Statins  Clopidogrel (Plavix) Surgical Procedures: 5 year risk of CVA 6%  Carotid endarterectomy or  Angioplasty & carotid stenting ( stenting can be considered in In symptomatic, highrisk candidates for carotid endarterectomy & In patients with surgically unapproachable stenosis)
  • 17. Intracerebral Hemorrhage (ICH)     In any patient with stroke symptoms, obtain NON-CONTRAST CT head first to r/o hemorrhagic stroke Most common cause is HTN. Other causes are amyloid angiopathy, vascular malformations, coagulopathy and cocaine abuse Clues for Amyloid angiopathy – primarily a lobar hemorrhage and seen mostly in elderly. Obtain cerebral angogram to rule out vascular malformations in :     Patients with ICH who are age under 45 Patients who developed ICH after cocaine use Rx  control BP to maintain SBP between 140 and 160 mm hg ( use IV labetalol or nitroprusside or nicardipine)  Mannitol and Hyperventilation to reduce intra cranial pressure In intracerebellar hemorrhages, realize that its very close to brain stem – so a mass effect can lead to Brain stem herniation. So, in intracerebellar hemorrhage, if extensive, will need URGENT SURGICAL DECOMPRESSION – contact neurosurgeon STAT!
  • 18.      Subarachnoid Hemorrhage Common causes : Berry aneurysms, AV malformations, Neoplasms C/F: Severe headache, photophobia, loss of consciousness, papilledema Prevention :  Obtain MRA to screen for Berry aneurysms in patients with ADPKD only if family member is diagnosed with an Intra Cranial Aneurysm or SAH, if the patient refers to symptoms related to an ICA or a patient has a high risk job  In patients with incidental aneurysms, if size > 10 mm  surgery. If size < 10mm, follow up MRI in 1 to 2yrs. Diagnosis : Non contrast CT first. If CT –VE, LP if suspicion is high  look for xanthochromia ( 10% SAH are missed by CT Scans) RX  If there is a ruptured aneurysm, rx with surgical clipping in 48 hours  Nimodipine to prevent post subarachnoid hemorrhage vasospasm and consequent, iscemic stroke ( Vasospasm after SAH is the most dreaded complication of SAH  leads to stroke)  Triple “H” therapy ( Induced Hypertension, hemodilution and hypervolemia) is also widely used in preventing and treating cerebral vasospasm after aneurysmal SAH.
  • 20. Meningitis    Symptoms : Fever, Photophobia, Headache, Neck stiffness, vomiting, seizures Use physical exam findings to confirm a diagnosis of meningitis. Look for:      Fever Nuchal rigidity Brudzinski's sign Kernig's sign Signs of encephalitis, such as weakness and change in mental status
  • 21. Meningitis  Do lumbar puncture to obtain CSF for:  Protein, glucose, and cell count determinations  Gram stain and bacterial culture  PCR testing for enterovirus and HSV if bacterial Gram stain and culture results are negative and cell counts suggest viral meningitis  Obtain CT scan before lumbar puncture in patients with: ( HIPFAN)  Immunucompromised state (I)  History of CNS disease (H)  New onset seizures (N)  Papilledema (P)  Altered level of consciousness ( suggests encephalitis)(A)  Focal neurologic signs (F) Be aware that delay in initiating appropriate antibiotics while awaiting results of CT scan in patients with bacterial meningitis may result in an adverse clinical outcome.
  • 23. Meningitis – Empiric Rx  Base empiric antibiotic therapy on:  Patient's age  CSF gram-stain result  Potential bacterial pathogens  Knowledge of local resistance patterns for those pathogens  Thereafter, base targeted antibiotic therapy on culture results and susceptibility data.  Administer dexamethasone 15 to 20 minutes before the first antimicrobial dose in adult patients with suspected meningitis.
  • 24. Meningitis – Emperical therapy Predisposing Factor AGE <1 month 1 - 23 months 2- 50 years >50 years Common Bacterial Pathogens Antimicrobial Rx Streptococcus agalactiae, Escherichia coli, Listeria monocytogenes, Klebsiella species Streptococcus pneumoniae , Neisseria meningitidis, S. agalactiae, Haemophilus influenzae, E. coli N . meningitidis, S. pneumoniae Ampicillin plus cefotaxime or ampicillin plus an aminoglycoside S. pneumoniae, N. meningitidis, L. monocytogenes , aerobic gram-negative bacilli Vancomycin plus a third-generation cephalosporin Vancomycin plus a third-generation cephalosporin Vancomycin plus ampicillin plus a thirdgeneration cephalosporin
  • 25. Meningitis – Emperical therapy Predisposing Factor Common Bacterial Pathogens Antimicrobial Rx   Basilar skull fracture S. pneumoniae, H. influenzae, group A -hemolytic streptococci Vancomycin plus a thirdgeneration cephalosporin Penetrating trauma Staphylococcus aureus, coagulase-negative staphylococci (especially Staphylococcus epidermidis), aerobic gramnegative bacilli (including Pseudomonas aeruginosa ) Aerobic gram-negative bacilli (including P. aeruginosa ), S . aureus , coagulase-negative staphylococci (especially S. epidermidis) Coagulase-negative staphylococci (especially S. epidermidis), S. aureus, aerobic gram-negative bacilli (including P. aeruginosa ), Vancomycin plus cefepime, vancomycin plus ceftazidime, or vancomycin plus meropenem  YOU ARE Adding an antipseudomonal antibiotic. HEAD TRAUMA Postneurosurgery CSF shunt Vancomycin plus cefepime, vancomycin plus ceftazidime, or vancomycin plus meropenem Vancomycin plus cefepime,c vancomycin plus ceftazidime,c or vancomycin plus meropenem
  • 26. Fall Prevention - Elderly
  • 27. FALLS IN ELDERLY Epidemiology  Falls occur in >30% of age over 65 years in community  Serious injury occurs in >20% of falls in older adults  Most falls occur in and around the patient's home Risk Factors for falls  Environmental hazards (most common)  Altered gait or balance  Lower extremity Muscle Weakness  Dizziness or Vertigo  Syncope  Postural Hypotension  Decreased visual acquity  Arthritis  Dementia or Altered Level of Consciousness  Major Depression  Medication use (especially more than 4 medications)  Class IA Antiarrhythmics, Digoxin, Diuretics , Anticonvulsants , Psychotropic medications like Benzodiazepines & Antipsychotics
  • 28. Screening and Evaluation for fall risk   Get Up and Go Test Cardiovascular exam   Postural Hypotension , Arrhythmias , Carotid Bruits Neurologic Exam Assess coordination and balance , Lower extremity muscle strength & Proprioception and vibration sense   Miscellaneous exam   Visual Acuity , Joint exam Diagnostics in cases of fall history        Complete Blood Count Thyroid Function TeSts Chemistry panel including Renal Function tests Serum Vitamin B12 Electrocardiogram Echocardiogram Brain Imaging
  • 29. Get up and Go Test Technique: Direct patient to do the following  Rise from sitting position  Walk 10 feet  Turn around  Return to chair and sit down Interpretation  Patient takes <20 seconds to complete test   Adequate for independent transfers and mobility Patient requires >30 seconds to complete test  Suggests higher dependence and risk of of falls
  • 30. Fall Risk - Prevention Use Assistive Devices  Wear flat, rubber soled shoes  Use ambulatory aid as needed (cane or walker)  Consider Hip protection device   Two convex shields worn inside underwear pocket Greatly reduces Hip Fracture Incidences Wearing pads: 0.39 Hip Fractures per 100 falls  Not wearing pads: 2.43 Hip Fractures per 100 falls   May reduce Incidence of pelvic fracture
  • 31. Fall Risk Prevention Education  Proper lifting technique  No stooping; bend knees and keep back straight Optimize Comorbid Conditions  Assess number/type of medications  Check Visual Acuity Vision <20/60 is a risk for falls Check for Cataracts Assess for depth perception    Control systolic Hypertension  Systolic Hypertension affects balance and fall risk  Avoid medications that increase fall risk  Medications causing hypotension or Dizziness Medications causing Sedation    Benzodiazepines   High risk of falls and Hip Fracture Highest risk within first 2 weeks of starting
  • 32. Fall Risk Prevention Modify home environment  Consider occupational therapy evaluation  Hand grips and safety mat in shower  Treads and handrails in stairway  Anchor rugs, non-skid rubber mats  Remove clutter, exposed wire, or cord  Keep halls and stairways well lit  Use nightlights in bathrooms and bedrooms Participate in regular Exercise (30 minutes, 4-5/week)  Walking Program  Exercise classes twice weekly reduces fall risk
  • 33. Headaches Migraines Tension headaches Cluster Headaches Temporal arteritis Chronic daily headache Analgesic Rebound Headache Post Traumatic Headache Menstrual Migraine Sinusitis
  • 35. Migraines - Features Presence of nausea or vomiting  Duration of typical untreated headache between 4 and 72 hours  Pulsatile or throbbing character  Unilateral location of head pain  Level of disability associated with typical attacks and impairment of usual daily activities ( any day during past 3 months)  Photophobia and phonophobia  Presence of at least 4 of the symptoms above has sensitivity of 29% for detecting migraine and specificity 100%  Presence of at least 3 of the symptoms above has sensitivity 80 and specificity 94 
  • 36. Migraine - Features  Consider using the pre-test probability of migraine and the number of symptoms in the POUND mnemonic to calculate the post-test probability of migraine.  In applying the mnemonic, determine whether the headache is:  P ulsatile in quality  Approximately O ne-day's duration (between 4 and 72 hours)  U nilateral in location  Accompanied by N ausea or vomiting  Of D isabling intensity
  • 37. Post-test Probabilities of Migraine Based on the Number of Migraine Features on History Compared to a Reference Standard of Headache Expert Neurologist Clinical Evaluation Result Pretest Probability (%)† 20 50 80 5 15 50 3 migraine symptoms* 75 94 99 4 or 5 migraine symptoms* 90 96 >99 0, 1, or 2 migraine symptoms*
  • 38. Migraine - Diagnosis    No need for neuroimaging for migraine patients and a normal neurologic examination Obtain imaging only if atypical headache features ( worst headache, headache awakening from sleep, headache Obtain an ESR in patients over age 50 who have new-onset headache to exclude the possibility of temporal arteritis.  Remember, however, that an elevated ESR (>30 mm/h) is not diagnostic of temporal arteritis and that a temporal artery biopsy is needed for diagnosis.
  • 39. Migraines - Rx   Identify and avoid dietary “triggers,” including: Caffeine withdrawal , Nitrates and nitrites in preserved meats, Phenylethylamines in aged cheeses, red wines, beer, champagne, chocolate, and monosodium glutamate in Asian and other prepared foods , Dairy products ,Fatty foods Recommend specific behavioral therapies for migraine patients, including relaxation training, biofeedback, or cognitive-behavioral therapy
  • 42. Drug Rx    Use migraine-specific agents (e.g., triptans, dihydroergotamine, ergotamine) - in patients with severe migraine - in those whose headaches have responded poorly to NSAIDs or combination analgesics such as aspirin plus acetaminophen plus caffeine. Triptans first choice because they are more effective and cause less nausea. All of these agents are contraindicated in the presence of CAD. (Ergots contraindicated, NSAIDS better avoided)
  • 43. Preventive rx Indications for daily preventive drug treatment in patients with significant disability related to frequent or severe migraine attacks (usually at least 2 per month):  Recurrent headaches that interfere with daily routine  Contraindication to acute (abortive) therapy , Failure or overuse of acute therapy , Adverse effects from acute therapy , A preference for preventive therapy  Choose among the following effective classes of agents, listed in order of strength of evidence for efficacy:  Non-selective, β-antagonists  Anticonvulsants : VALPROIC ACID, TOPIRAMATE  Be aware that valproate is the only antiepileptic drug approved by the FDA for migraine  Antidepressants, calcium antagonists, ARB ( Candesartan)  Consider perimenstrual preventive treatment with a triptan for menstrually associated migraines. ( also ocpills without off period, Seasonale etc).  Try a drug therapy for at least 2 months before changing the agent.  Consider tapering or discontinuing preventive treatment after a sustained reduction in headache frequency that lasts 6 to 12 months.
  • 44. Referral Refer the following to a migraine specialist:  Intractable migraine (status migrainosus)  Medication-overuse headache or chronic migraine (sometimes described as “rebound” headache, transformed migraine, or chronic daily headache)  Analgesic dependency (especially narcotic analgesics)
  • 46. Cluster Headaches        Excruciating headache pain Duration of typical attack - 15 to 180 minutes Symptom-free intervals vary from 1 to 48 hours Frequency : up to 8 attacks/day Headache attacks come in clusters daily or at variable periods. Nausea, photophobia and phonophobia Tends to occur more in nights- The headaches have a predilection for the first rapid eye movement (REM) sleep phase so the cluster patient will awaken with a severe headache 60 to 90 minutes after falling asleep Look for presence of autonomic symptoms : Tearing, Conjunctival injection, Ptosis , Rhinorrhea , Nasal congestion , Forehead and facial sweating and Miosis  Note the behavior during attacks (usually agitated, unlike in migraine where the patient is quiet and withdrawn!) ----------edited ---
  • 47. Cluster Headache - Physical During an attack, look for autonomic features that will help you diagnose this headache:  Ipsilateral ptosis  Miosis  Eyelid edema  Lacrimation  Rhinorrhea  Nasal congestion  Forehead and facial sweating  Conjunctival injection ANY OF ABOVE CLUES AT THE TIME OF HEADACHE CAN HELP IN DIAGNOSIS 
  • 48. Investigations Cluster headache is a clinical diagnosis.  Obtain neuroimaging with MRI and MRA in patients with:  Atypical headache features OR Abnormalities on neurologic examination OR Lack of response to previous treatments   Order an ESR to look for temporal arteritis in patients over age 50 with headache and visual loss.  Obtain a polysomnogram in patients with headache to identify sleep-disordered breathing (sleep apnea), especially those with snoring or daytime sleepiness. 
  • 49. Treatment Identify and recommend that patients avoid trigger factors of cluster headache such as: Alcohol , Smoking , Solvents , Altitudes above 5000 feet , High temperatures , Altered sleep schedules, e.g., afternoon naps, work shift changes, traveling across time zones, delay or advanced sleep phase  Acute Rx : Give 100% oxygen administered by nonrebreather face mask at 7 to 15 L/min for 15 to 30 minutes and repeat as needed.  To treat acute attacks, consider using separately and then combine as necessary:  Subcutaneous or intranasal sumatriptan, or nasal zolmitriptan  Intramuscular, subcutaneous, or intravenous DHE  Oral or rectal ergotamine  Other options : oral corticosteroids, Ipsilateral occipital nerve block 
  • 51. Clinical features       Pain lasts from few seconds to minutes. “Electric” like in character. Can be spontaneous or triggered by light touch, talking or eating. Distributed trigeminally (usually second or third divisions), either alone or in combination First division pain around the eye or forehead occurs in 10%-20% of patients, often with pain in other parts of the face, usually mid-cheek and upper lip or teeth Usually unilateral Pain is relieved by carbamazepine or oxcarbazepine but not narcotics or milder analgesics  Consider using carbamazepine administration as a diagnostic maneuver
  • 52. Clinical Features   Refractory period of pain after stimulation of the trigger area (cannot elicit pain again by touching or pushing immediately after a painful attack)  Highly predictive of trigeminal neuralgia. Neurological exam is usually normal  Note that abnormal neurological findings are indicative of diseases other than trigeminal neuralgia.
  • 53. Treatment  Avoid PAIN triggers  Stay away from the direct blast of an air conditioner , Cover the face before going out into a cold wind, using covers that do not touch the face or that are not tight fitting , Avoid foods that trigger the pain for an individual patient (e.g., hot or cold drinks, foods that require much chewing)
  • 54. Treatment     Mild, infrequent pain  no treatment Drug of choice for Trigeminal neuralgia  Oxcarbazepine or Carbamazepine If pain persists, add another drug, such as gabapentin, baclofen, lamotrigine, or pregabalin If pain continues despite drug therapy  needs neurosurgical intervention  gamma knife radiosurgery or percutaneous balloon microcompression of trigeminal division that is causing symptoms.
  • 56. Tension Headaches        Most common form of headache Last from 30 minutes to 7 days Typically have bilateral location Have a nonpulsating pressing or tightening quality described by patients as a “band-like” constriction around their head, Are mild to moderate in intensity Do not prohibit activity. ( Unlike migraines) There is no aggravation of headache by using stairs or by doing any similar routine activity; not associated with nausea or vomiting (although anorexia may occur); neither associated with both photophobia and phonophobia (but may exhibit one or the other)
  • 57. Treatment      Try Acetaminophen first NSAIDS if no response to tylenol. OTC NSAIDS and then prescription NSAIDS Combination Medications: Aspirin or acetaminophen (or both of these analgesics) are often combined with caffeine or a sedative drug in a single medication. For example, Excedrin combines aspirin, acetaminophen and caffeine  Combination drugs such as this may be more effective than are pure analgesics for pain relief. Many combination drugs are available over-the-counter, analgesicsedative combinations can be obtained only by prescription because they may be addictive and can lead to chronic daily headache. Advise pts not to use these drugs more than two days a week, as they can lead to chronic daily headache Opiates, or narcotics, are rarely used because of their side effects and potential for dependency. These include codeine combined with acetaminophen
  • 58. Chronic Daily Headache Rebound Headaches Analgesic overuse headaches Chronic Migraine
  • 59. CDH    Chronic daily headache" (CDH) includes a variety of headache types, of which chronic migraine is the most common. Medication overuse, or drug rebound headache, is the most treatable cause of refractory daily headache. A pathologic underlying cause should be considered in patients with recent-onset daily headache, a change from a previous headache pattern, or associated neurologic or systemic symptoms.
  • 60. CDH – d/d Primary Headaches Headache Duration > 4hrs Headache Duration < 4hrs Chronic (transformed) migraine Strictly unilateral-prominent autonomic features(SUNCT) Chronic tension-type headache Cluster headache New daily persistent headache Paroxysmal hemicrania Hemicrania continua Trigeminal neuralgia Cough headache Benign exertional headache Headache associated with sexual activity
  • 61. CDH – d/d Secondary Headaches                 Headache associated with vascular disorders Arteriovenous malformation Giant cell arteritis Carotid dissection Vasculitis Headache associated with nonvascular intracranial disorders Neoplasm Idiopathic intracranial hypertension (pseudotumor cerebri) Infection Post-traumatic headache Subdural hematoma ( chronic) Myofascial pain Cervical spine disorders Temporomandibular joint dysfunction Headache caused by sleep disorders Obstructive sleep apnea – early morning headache after sleep
  • 62.      CDH Most patients with CDH have chronic (transformed) migraine. There is a hx of episodic migraine that has evolved (transformed) over time into a pattern of almost daily headaches. These daily headaches may be mild, but migraine flares may continue to be superimposed on the daily headache symptoms. The most common causes of migraine transformation  a)frequent headaches at baseline and b) obesity. Other modifiable risk factors for transformation include medication overuse, snoring, and stressful life events.
  • 63.        Drug rebound & Medication overuseto improve despite use of Patients who do not stop analgesic overuse fail preventive therapy  patients who stop taking analgesics on a daily basis have a marked reduction in frequency of headache Drug rebound headache is a common treatable cause of transformed migraine. Patients who have drug rebound headache are refractory to usual acute and prophylactic interventions  CLUE : The patient who repeatedly presents to the emergency department requesting narcotics for headache relief most commonly has drug rebound headache. The sustained use of these medications more than three days per week is sufficient to develop drug rebound headache. All headache medications, including triptans,have the potential to cause drug rebound headache. Most common agents that cause drug rebound headache are narcotics, butalbital products, and combination products containing caffeine. Be alert to recognize signs of secondary headache in patients who are selfmedicating frequently  Only after a careful evaluation for secondary headache should drug rebound headache be suspected in patients with medication overuse.
  • 64. Clinical Approach in Patients with Chronic Daily Headache 1.Treat medication overuse, if present ( next slide) 2.Select pharmacologic therapies 3.Treat potential underlying pathology  Myofascial pain, Physical therapy, Temporomandibular treatment, Psychiatric comorbidity (antidepressants, anxiolytics) and Sinus evaluation and treatment etc 4.Limit symptomatic medication use to two days per week (after withdrawal ["detoxification"] is completed). Recommend use of nonsteroidal anti-inflammatory drugs. Recommend use of triptans for migraine flares. Avoid use of medications prone to drug rebound, especially combination analgesics, caffeine-containing compounds, butalbital products, and narcotics. 5.Consider behavior therapy Encourage lifestyle management, Regular exercise, Regular meals; no caffeine; migraine diet Sleep hygiene Stress reduction Biofeedback Cognitive-behavior therapy 6.Monitor progress (using headache calendar).
  • 65. Treatment of Medication Overuse (Drug Rebound Headache) 1.Withdrawal of symptomatic medications, including caffeine. A.Gradually taper medications in patients where physiolgic withdrawal is a concern (i.e., narcotics, butalbital). B.Use abrupt withdrawal or taper medications in all other patients. 2.Preventive therapy Any preventive therapy, or combination medicine for headache 3.Transition therapy Daily migraine-specific therapy Dihydroergotamine (DHE), intranasal, intramuscular, or intravenous Long-acting triptan: naratriptan (Amerge) or frovatriptan (Frova) B.Anti-inflammatory agents Short course of corticosteroids Long-acting nonsteroidal anti-inflammatory drugs 4. Rescue therapy, as needed A.Non-narcotic analgesics: parenteral ketorolac (Toradol) B.Antiemetics C.Sedating antihistamines: diphenhydramine (Benadryl) or hydroxyzine (Atarax)
  • 67. Pseudotumor Cerebri        Female to male ratio = 8:1 Females in reproductive age group – common Obesity and recent weight gain - an important risk factor. Pregnancy can precipitate Commonest complaint – headache. Can cause vision loss if papilledema is severe.  there is no altered level of alertness, cognitive impairment, or focal neurological findings that are usually associated with the elevated ICP Physical : papilledema Diagnosis : CT head, Lumbar puncture – elevated opening pressure. Symptoms improve after LP which is highly suggestive of BIH. Send CSF to all studies to r/o other etiologies
  • 68. Treatment Medical Rx  Weight loss ( goal 1lb/week for 2 months or longer)  Low sodium diet  Diuretics : Acetalozamide, Furosemide Surgical Rx: Optic nerve fenestration Lumbo-subarachnoid Peritoneal shunting
  • 70. Bells Palsy Because Bell's palsy is a diagnosis of exclusion, make every effort to exclude other identifiable causes of facial paralysis Consider the following:  Lyme disease : Obtain Lyme disease testing in patients with acute facial paralysis in endemic areas (Obtain an ELISA serology for B. burgdorferi in endemic areas if exposure is likely  Obtain confirmation by Western blot testing if the ELISA result is positive. )  Otologic disease ( chronic otitis media, cholesteatoma)  Neoplasm  Neurologic diseases, such as stroke and MS 
  • 71. Bells Palsy – Clues in Hx          Rapidity of onset  Onset over more than 3 weeks strongly suggests neoplastic origin Hearing loss  Hearing loss is not usually associated with Bell's palsy  Sensorineural hearing loss suggests Ramsay Hunt syndrome, infectious or neoplastic processes in the middle ear, cerebella pontine angle affecting the brain stem.  Conductive hearing loss suggests infectious/neoplastic processes within the temporal bone, especially otitis media/mastoiditis or cholesteatoma Periauricular pain  Mild to moderate pain is common for several days before or after onset of paralysis. Severe persistent pain suggests Ramsay Hunt syndrome or neoplasm Bilateral facial palsy  Unlikely to be Bell's palsy. Consider Lyme disease, Guillain-Barré, sarcoidosis, infectious mononucleosis, myasthenia gravis, botulism, acute porphyria, amyloidosis Decreased tearing or salivary flow  Common in Bell's palsy secondary to involvement of parasympathetic fibers that travel with the seventh nerve to the lacrimal and salivary glands Dysgeusia  Common in Bell's palsy due to involvement of the chorda tympani Duration of paralysis  Complete paralysis persisting beyond 6 months. There is always some recovery from Bell's palsy. Absence of any signs of return of function suggests a neoplastic etiology Eye discomfort  A common and important problem in Bell's palsy. It is due to poor corneal hydration and corneal drying secondary to diminished lacrimation and absent blink reflex Decreased visual acuity  A common and important problem in Bell's palsy. It is due to poor corneal hydration and corneal drying secondary to diminished lacrimation and absent blink reflex
  • 72. Bells Palsy – Clues in Physical     A complete ear exam is important to exclude otologic reasons for seventh nerve paralysis. A maculopapular or vesicular rash in external ear canal suggests Ramsay Hunt syndrome Neurologic exam for segmental focal paralysis  Less than the entire half of the face is involved; e.g., involvement of only the mouth or only the eye  Neoplasm of parotid most likely etiology, rather than Bell's palsy, especially if associated with persistent pain Neurologic exam for forehead-sparing facial paralysis  Indicates CNS etiology. Consider stroke ( UMN Lesion) Neurologic exam for balance disturbance  Consider other neurologic disorder or Ramsay Hunt syndrome rather than Bell's palsy. Ramsay Hunt syndrome produces balance disturbance by involvement of the vestibular nerve and/or semicircular canals
  • 73. Treatment      Prescribe a short course of high-dose prednisone (1 mg/kg·d to 70 mg/d for 7 days, then tapered) for Bell's palsy as soon as the diagnosis is made, provided that the patient is seen within 1 week of the onset of paralysis, and regardless of the degree of paralysis. Consider antiviral therapy: Acyclovir, 200 mg orally, 5 times per day Valacyclovir, 500 to 1000 mg orally, three times per day Famciclovir, 500 mg orally, three times per day
  • 74. Treatment Manage dry eye aggressively to avoid ophthalmologic complications  Until complete eye closure and a good blink reflex return, patch the eye at night, and ensure that the patch itself does not touch the cornea and abrade it, Prescribe artificial tears as needed (as often as every 15 minutes) during the day, and an ocular lubricant at night.  Refer to physical therapist :  If loss of facial movement is incomplete  After recovery of movement has begun 
  • 75. Follow-up Identify patients who experience atypical recovery or develop treatment side effects.  Reevaluate the patient within the first 4 weeks.  Evaluate the patient about every 3 months thereafter.  Instruct the patient to call immediately if any of the following develop:    Persistent eye pain Hearing loss or vertigo New neurologic signs or symptoms
  • 77. Symptoms n Signs              Sensory loss or paresthesias Partial or complete monocular loss of vision associated with pain (optic neuritis) Blurred vision or impaired color vision Motor weakness suggesting myelitis syndrome, especially paraparesis Imbalance due to ataxia or sensory ataxia Spinal or limb paresthesias elicited by neck flexion (Lhermitte's sign) Bladder or bowel dysfunction such as urgency, incontinence Diurnal fatigue Heat sensitivity, causing worsening of fatigue or neurologic symptoms Memory loss or other cognitive symptoms Diplopia Facial pain consistent with trigeminal neuralgia Dysarthria or dysphagia
  • 78. M.S Classification Using the neurologic history, determine if the subtype of MS is:     Relapsing remitting: clinical onset occurs with a “relapse” (“attack”; “exacerbation”; “flare-up”) of neurologic symptoms or signs that fully resolve or leave minimal residual deficit Primary progressive: clinical onset is insidious and worsening of symptoms occurs gradually over months to years Secondary progressive: sustained gradual worsening of baseline status, even between relapses, in patients who initially had relapsing-remitting disease Progressive relapsing: a primary progressive onset but the later establishment of discrete clinical relapses
  • 79.    Obtain: Brain MRI to assist with MS diagnosis and to exclude coexisting or other neurologic disorders Spinal cord MRI if needed to show dissemination in space or to exclude a compressive lesion
  • 80.    Do a lumbar puncture when appropriate based on 2005 Revised McDonald Criteria, usually when an MRI does not establish dissemination of white matter lesions in space. Consider a positive result for purposes of 2005 Revised McDonald Criteria to be presence of oligoclonal IgG bands in CSF and not in serum, or an elevated IgG index. If MS cannot be confirmed and another diagnosis is not made, consider serial clinical follow-up examinations and repeated MRI testing to detect clinical or radiological changes that may indicate MS.
  • 81. Revised McDonald Criteria for Diagnosis of MS
  • 82. Clinical (Attacks) Objective Lesions Additional Requirements to Make Diagnosis 2 or more 2 or more None; additional evidence desirable but must be consistent with MS 2 or more 1 Dissemination in space by MRI or positive CSF and 2 or more MRI lesions consistent with MS or further clinical attack involving different site 1 2 or more Dissemination in time by MRI or second clinical attack 1 (monosympto matic) 1 Dissemination in space by MRI or positive CSF and 2 or more MRI lesions consistent with MS AND Dissemination in time by MRI or second clinical attack 1 (progression from onset) 1 One year of disease progression (retrospectively or prospectively determined) AND Two of the following: a) Positive brain MRI (nine T2 lesions or four or more T2 lesions with positive visual evoked potentials) b) Positive spinal cord MRI (two focal T2 lesions) c) Positive CSF Definition of positive MRI—3 out of 4 of the following: 1 gadolinium-enhancing lesion or 9 T2 hyperintense lesions if no gadolinium-enhancing lesion; 1 or more infratentorial lesions; 1 or more juxtacortical lesions; 3 or more periventricular lesions. Note: A spinal cord lesion can be considered equivalent to an infratentorial lesion in the brain. Thus, an enhancing spinal cord lesion is considered to be equivalent to an enhancing brain lesion, and individual spinal cord lesions can contribute together with individual brain lesions to reach the required number of T2 lesions.
  • 83. MS - Admission Hospitalize patients with:  Progressive, rapid, or severe deterioration in bulbar dysfunction (especially swallowing), ambulatory function, or level of consciousness  Medical disorders requiring inpatient evaluation and therapy, such as infections or pressure ulcers
  • 84. Treatment   Use an immunomodulatory therapy (βinterferon preparation or glatiramer acetate) in patients with RRMS to reduce rates of clinical relapse and prevent new MRI lesions. Natalizumab for patients with established relapsing MS who respond inadequately to or cannot tolerate other approved MS immune therapies to delay worsening of physical disability and reduce the frequency of clinical exacerbations.
  • 85. Treatment        Consider the use of interferon-β for patients with secondary progressive MS Initiate corticosteroid therapy such as methylprednisolone, 1000 mg/d iv for 3 to 5 days, in patients with exacerbations of MS causing functional impairment Treat refractory MS-related fatigue with amantadine, 100 to 200 mg/d; aspirin, 1300 mg/d; or modafinil, 200 mg/d. Treat appropriately any other medical disorders or clinical situations that might worsen MS symptoms or signs such as systemic infection, a worsening or new medical problem, or medication change Prescribe an antispasticity drug such as baclofen, tizanidine, or diazepam for patients with muscle spasms or excessive spasticity interfering with function. Prescribe tricyclic antidepressants such as amitriptyline or nortriptyline, with or without analgesics, for patients with constant epicritic (prickling, burning) neuropathic pain. Prescribe anticonvulsant medications such as carbamazepine, oxcarbazepine, or gabapentin for patients with trigeminal neuralgia or similar paroxysmal neuropathic pain (shooting, jabbing pain) associated with MS.
  • 86. Follow-Up     Determine if the patient is experiencing significant and unacceptable side effects and try to minimize them, for example: In patients on corticosteroid therapy, monitor blood pressure and dietary and blood sugar issues In patients on interferon therapy,  Monitor for injection-site reactions, skin necrosis, depression or suicidal ideation, flu-like symptoms, and obtain laboratory tests every 6 months:  CBC with differential  AST  ALT  Alkaline phosphatase  In patients using any of the β-interferon products, consider measurement of serum neutralizing antibody status at 12 months and at 24 months after starting therapy or at the time of a clinical relapse In patients on immunosuppressive therapy, avoid live vaccines and unnecessary exposure to viral illnesses
  • 88. Symptoms N Signs Symptoms  Ptosis or diplopia – esply, new onset  Painless difficulty with repetitive tasks  Painless difficulty in climbing stairs, getting up from a chair, or walking  Painless difficulty holding arms over head  Dyspnea or difficulty swallowing or chewing  Difficulty holding the head up, or neck muscle soreness Signs:  Ptosis  Extraocular muscle weakness  Facial weakness  Difficulty swallowing  Effort-related dysarthria  dysarthria that worsens with continued speech.  Proximal muscle weakness  Neck weakness
  • 89.         Diagnosis Anti-Ach Receptor antibodies are diagnostic. Anti-AchR antibodies are 50% sensitive and 99 specific Anti-Musk antibodies are 40% sensitive and 99% specific. In patients with generalized myasthenia who are seronegative for anti-AChR antibodies, obtain anti-MuSK antibodies. In patients with purely ocular signs of myasthenia who are negative for anti-Musk and Anti-AchR –ve, diagnose with pharmacologic and electrophysiologic tests ( Edrophonium test, EMG)  the sensitivity of EMG is 98% and specicficity is 50-90%. Consider this diagnosis in presence of classical clinical features even in the absence of antibodies. CT chest to r/o thymoma in a pt with newly diagnosed myasthenia R/O common co-existent conditions – Hashimato’s thyroiditis, DM
  • 90. Diagnosis   If all tests show negative results  Consider additional neurophysiologic, serologic, neuroimaging, CSF, genetic, and muscle biopsy testing If no other diagnosis can be made, consider repeated myasthenia gravis testing in future
  • 91. Differential Diagnosis      MS  History or physical findings of autonomic, special sensory (visual acuity or visual fields, hearing), or sensory involvement is classic in MS and is not seen in myasthenia ALS  Progressive course, symptoms and signs of both upper and lower motor neuron disease, asymmetry of weakness early in the disease course, and fasciculations and lack of involvement of eye movement, sensory or autonomic, are classic for ALS  Unless there is an independent cause of diplopia or abnormalities of eye movement, the presence of these is essentially incompatible with ALS  Distinction from myasthenia is particularly difficult for bulbar ALS, but the presence of significant tongue atrophy and/or fasciculations, or the presence of signs of upper motor neuron dysfunction (brisk jaw jerk) suggest the diagnosis of motor neuron disease ( ALS) Hypothyroidism  obtain TFTs Botulism  Descending progressive Paralysis Eaton-Lambert Syndrome  50%-60% pts have small cell lung ca. Repetitive stimulation studies cause improvement and this distinguishes this disorder from myasthenia gravis
  • 92. Treatment          Ach esterase inhibitors for symptoms  Pyridostigmine, Neostigmine Consider immunotherapy based on the course of an individual patient  treat with one or more immunosuppressive or immunomodulatory therapies including: Corticosteroids such as prednisone or prednisolone Azathioprine Mycophenolate mofetil Cyclosporine Cyclophosphamide Plasma exchange Intravenous immunoglobulin IN MYASTHENIA CRISIS
  • 93. Treatment   Consider modifying the extent of different types of physical activity including: Heavy physical effort , Prolonged speaking , Diet (soft rather than regular) & Prolonged reading Recommend thymectomy in all patients with evidence of thymoma on chest CT or MRI unless extensive local spread has already occurred.
  • 94. Drug induced Myasthenia  Penicillmine, Aminoglycosides can cause drug-induced myasthenia  early diagnosis and discontinuation of the drug will prevent worsening of symptoms
  • 95. ALS
  • 96. Symptoms         Weakness of gradual onset , Muscle wasting , Muscle twitching ( suggests LMN lesion), Muscle cramps , Clumsiness , Muscle stiffness ( UMN impairment), Changes in voice or articulation , Difficulty swallowing , Difficulty breathing ( bulbar involvement) Spread of symptoms within one region or to additional regions defined as cranial, cervical, thoracic, and lumbo-sacral  gradual progression of symptoms suggesting a degenerative disease Confirm the absence of symptoms that are not typical for ALS, including: Predominant sensory symptoms ( favors demyelinating diseases, neuropathy) Pain as predominant symptom ( favors radiculopathy, myelopathy) Bowel or bladder incontinence ( favors cervical myelopathy, demyelinating diseases) Cognitive impairment Ocular muscle weakness ( favors myasthenia)
  • 97. Signs Muscle wasting and weakness  Fasciculations  triggered by muscle contraction or tapping on a muscle  Spasticity  suggests UMN involvement  Hyperreflexia  Hyporeflexia  Brisk jaw jerk/ gag reflex  Evaluate for respiratory muscle weakness looking for  Use of accessory muscles  Paradoxical breathing  Low vocal volume  Lack of breath support  Tachypnea 
  • 98. Diagnosis   EMG : Look for electromyographic evidence of active and chronic denervation in at least two of four regions of the CNS ( Brainstem, cervical spinal cord, thoracic spinal cord, lumbosacral spinal cord) Do neuroimaging to r/o other conditions.
  • 99. Treatment Offer noninvasive ventilatory support to patients with ALS and respiratory insufficiency.  Monitor respiratory status in ALS patients:     Consider noninvasive ventilatory support:         Do serial FVC or sniff nasal pressure measurements Ask about symptoms of respiratory insufficiency such as insomnia, daytime fatigue, morning headaches Obtain overnight pulse oximetry or polysomnography as needed In patients with symptoms of respiratory insufficiency When FVC (sitting or supine) is at or below 50% of predicted normal If there is evidence of nocturnal hypoventilation (O2 saturation <90%), even if sitting and supine FVC are not significantly reduced Consider mechanical insufflation-exsufflation for airway secretion management in patients with insufficient cough. Consider a portable suction machine for secretion management. Physical therapy Occupational therapy Speech therapy
  • 100. Treatment Prescribe riluzole, 50 mg po bid. Use caution with riluzole and any of its components in patients who:  Have abnormal liver function , Have renal insufficiency , Are elderly ,  Monitor for adverse effects of riluzole treatment, Abdominal pain, anorexia, diarrhea , Arthralgia, asthenia       Nausea, vomiting Hypertension, tachycardia (rare) ALT elevations (rare) Jaundice (rare) Neutropenia (rare) Obtain LFT and blood counts every month for the first 3 months, then every 3 months for the first year, and periodically thereafter while the patient is on riluzole.
  • 101. Neurosyphilis Refer to ID slides under “Syphilis”
  • 104. Risk Factors for Dementia            Age Family history of dementia History of hypertension History of head injury Low education attainment (<10 years) Alcohol abuse Current ASA use Pesticides and fertilizers Liquid plastics or rubber ? Screen patients  Because the incidence of dementia increases with age, screening has generally been considered for people over a certain age, usually 60 or 65, but there is currently no evidence on which to base such recommendations Screening test – MMSE – Score of 26 or lower is dementia
  • 105. Diagnosis Ask the patient and the family member about:  Memory loss  Getting lost  Word-finding difficulties  Impaired ADL, such as dressing, grooming, and housework.  Changes in:  Personality , Mood m Energy , Appetite , Sleep , Enjoyment of activities ( Depression is an important differential diagnosis)  Behavioral changes, including:  General activity level , Eating , Drinking , Sleep , Sex  Neurologic deficits, including:  Gait abnormalities  Falls  Weakness  Clumsiness  Sensory abnormalities  Abnormal movements  Incontinence
  • 106. Diagnosis   Nature and time course of cognitive problems, especially acute vs. subacute course, or evidence of fluctuating level of consciousness. The sequence in which the cognitive difficulties and other symptoms developed should be chronicled  An acute change suggests delirium or a recent CNS event  Fluctuating level of consciousness suggests delirium.    Temporal association with a change in medication suggests a causal relationship. Classic Alzheimer disease presents with early loss of short-term memory, language, and visuospatial abilities, but preserved personality and normal neurologic exam Personality change  May suggest frontal lobe pathology (e.g, stroke, tumor, or frontotemporal dementia)
  • 107. Physical exam Do a mental status exam to evaluate:  Level of alertness  Short- and long-term memory  Orientation  Concentration  Abstract reasoning  Language (naming, vocabulary, fluency, repetition, comprehension)  Visuospatial abilities (clock drawing, design copy)  Cortical-sensory integrative function (neglect, left-right differentiation, stereognosis, and graphestesia)  Praxis  Mood  Hallucinations or delusions  Apathy  Do a comprehensive neurologic exam, including cranial nerve, motor, sensory, reflex, and cerebellar function, to look for concurrent CNS disease.
  • 108. D/D = Dementia                   Delirium Drugs ( esply BZDs) Depression Alzheimers Vascular dementia ( step wise deterioration) Lewy body dementia ( Parkinsons disease + visual hallucinations) NPH Sub dural hematoma Fronto temporal dementia – including picks disease ( personality changes) Vitamin b12 deficiency Traumatic brain injury Thyroid disease Brain tumor HIV dementia Huntington disease Parkinson disease Chronic alcohol use CJD
  • 109. Vascular dementia      Stepwise” deterioration. Loss of function should be correlated temporally with cerebrovascular events. Level of consciousness must be normal to make the diagnosis. May also be present in patients with “silent” strokes, multiple small strokes, or severe diffuse CVD. Should be suspected in any patient with cerebrovascular risk factors, even if a neurologic exam doesn’t suggest a stroke
  • 110. CJD       Rapid progression Early age of onset Prominent myoclonus Characteristic EEG pattern of triphasic sharp waves (1-2 Hz) Diffusion-weighted MRI may be more sensitive and specific for the diagnosis of this condition Poor prognosis
  • 111. HIV Dementia    Seen with advanced HIV Memory disturbance usually accompanied by lethargy and social withdrawal, as well as by motor dysfunction (ataxia, weakness, and incoordination). Aphasia, apraxia, and agnosia are rare
  • 112. NPH       Clinical triad of dementia, gait abnormality (slow, broad-based, impaired turning), and urinary incontinence. Dementia is often associated with psychomotor slowing and apathy. Dementia and apathy may be the earliest symptoms CT scan of head is useful. If suspicion is high, lumbar puncture with pregait and post-gait monitoring is done. Ventriculo-peritoneal shunting can be curative in some patients
  • 113. Delirium vs. Dementia       Altered level of alertness and attention, often in conjunction with globally impaired cognition. Onset may be abrupt, and fluctuating level of alertness is common. Older patients often appear to have psychomotor retardation, and may show the full range of mental status abnormalities, including depressed or elevated mood, hallucinations, delusions, and agitated behavior Delirium must be excluded in order to diagnose dementia. Making the diagnosis is critical, because it often reflects a serious systemic disturbance. Metabolic derangement, medication effects, and infection are the most common causes
  • 114. Alzheimer’s Disease        Gradual memory loss, preservation of level of consciousness, inefficiency or impairment in ADL performance may also be present. May initially become manifest when the patient has lost a significant source of assistance, for example the loss of a spouse or a significant change in routine (e.g., moving). Neurologic signs, such as falls, tremor, weakness, or reflex abnormalities, are not typical early in the disease course. As the illness progresses, other cortical deficits, such as aphasia, apraxia, agnosia, inattention, and left-right confusion will develop. Seizures are present frequently in advanced disease; their presence earlier in the course suggests a diagnosis other than Alzheimer disease. The presenting symptom to the physician may not be a cognitive problem. Often, the earliest presenting symptoms are paranoid delusions or depression, which upon further investigation turn out to be part of a dementia
  • 115. Alzheimers - diagnosis       Criteria for clinical diagnosis of PROBABLE Alzheimer's disease include: Dementia established by clinical examination and documented by the Mini-Mental Test, Blessed Dementia Scale, or some similar examination, and confirmed by neuropsychological tests: Deficits in two or more areas of cognition Progressive worsening of memory and other cognitive functions No disturbance of consciousness Onset between ages 40 and 90, most often after age 65, and absence of systemic disorders or other brain diseases that in and of themselves could account for the progressive deficits in memory and cognition
  • 116. Drug therapy - Dementia      Use acetylcholinesterase inhibitors to delay cognitive decline in Alzheimer disease, dementia with Lewy bodies, and mixed Alzheimer disease and vascular dementia Use donepezil, rivastigmine, or galantamine to delay progression of symptoms in Alzheimer disease, dementia with Lewy bodies, and mixed Alzheimer disease and vascular dementia. Initiate treatment once the diagnosis has been made and the patient is medically and psychiatrically stable. Ensure that treatment is continuous and without lengthy interruptions. Increase medication doses monthly until target doses are reached.
  • 117. Drug therapy - Dementia     Do not prescribe high-dose vitamin E routinely to slow the progression of symptoms in Alzheimer disease. Use memantine to delay cognitive decline in moderate to advanced Alzheimer disease and vascular dementia. , Begin with 5 mg/d.  Increase medication doses weekly by 5 mg/d until the target dose of 10 mg twice daily is reached.  Add memantine in patients on a stable dose of a cholinesterase inhibitor, but do not use it as a substitute Consider using Ginkgo biloba extract, 120 to 240 mg/d, in patients with mild to severe Alzheimer disease, but recognize that there are insufficient data to recommend its use. Replace vitamin B12 in patients with evidence of tissue deficiency of vitamin B12 (elevated methylmalonic acid and homocysteine).
  • 118. Drug Therapy Treat psychotic symptoms or behavioral disturbances complicating dementia with drugs  Consider using one of the following antipsychotic medications in the treatment of psychotic symptoms (hallucinations and delusions) or behavioral disturbances (aggression, severe irritability, agitation, explosiveness) if there is a risk of harm to the patient or others, or if patient distress is significant and non-drug treatments have been ineffective: Olanzapine:, Risperidone: Quetiapine, Acetylcholinesterase inhibitors (donepezil, galantamine, or rivastigmine)  Minimize use of antipsychotics in patients with dementia to the extent possible by:  Using the lowest effective dose  Restricting use of antipsychotics to patients with hallucinations, delusions, or agitation, the symptoms for which these medications have proven efficacy  Treating these symptoms only if they are causing significant problems, such as distressing the patient or caregiver, jeopardizing a living arrangement, or necessitating psychiatric hospitalization  Reevaluating the need for continued antipsychotic use on a regular basis  Consider consultation with a geriatric psychiatrist in patients with difficult to treat symptoms or the development of polypharmacy 
  • 119. Drug therapy - Pseudodementia      Treat patients with significant symptoms of depression with antidepressant drugs: Sertraline, Paroxetine, citalopram, fluoxetine, venlafaxine Avoid agents with prominent anticholinergic effects, such as amitriptyline and imipramine, in patients with dementia Maintain treatment at therapeutic doses for at least 6 to 8 weeks before declaring the trial a failure. Consider referral to a geriatric psychiatrist if one or two trials of antidepressants (at therapeutic doses given for at least 6 to 8 weeks) have failed or cannot be tolerated due to side effects. Be aware that all of these drugs can cause or exacerbate delirium.
  • 120. Driving - Dementia  Address the patient's driving ability with the patient and caregiver.       Inquire about motor vehicle accidents or near accidents, and changes in driving habits or patterns. Advise patients who already show driving impairment that for their own safety and the safety of others they must no longer drive. Advise patients who have received the diagnosis of dementia, but have not yet shown any difficulties with driving, to undergo a driving evaluation and to refrain from driving before completion of the evaluation. Note that driving evaluations are usually available at the local motor vehicle agency or hospital departments of occupational therapy. Follow state law with regard to informing the motor vehicle agency of a patient's impaired driving ability. For patients who are able to continue to drive, repeat driving assessment every 6 months.